You are on page 1of 58

Chapter 1

Power System Generation,


Transmission and Distribution
LEARNING OBJECTIVES

After reading this chapter, you will be able to understand:


• Basics of power system • Types of boilers
• Structure of power systems • Types of super heaters
• Basic power generation concepts • Types of condensers
• Thermal station • Types of hydro power stations
• Coal and ash handling plant • Mass curve

BaSicS of Power SyStem The term containing sinθ component of the instantaneous
power ‘S’ is called the ‘instantaneous reactive power’. The maxi-
Complex Power mum value of the pulsating power, designated Q is called reac-
The total power in a single-phase AC system at any instant is the tive power or reactive volt ampere. The units for Q is VARS (Volt
product of the instantaneous voltage and instantaneous current. Ampere reactive), the reactive power is
For a single-phase AC circuit, the voltage and current phasors are V I
conventionally Q = max max sin θ
2
written as
Q = V I sin θ
uan = Vmax coswt
The square root of the sum of the squares of P and Q is equal to the
ian = Imax cos(wt – θ) product of |V| and |I|.

(V I cos θ ) + ( V I sin θ ) = V I
2 2
The instantaneous power is P2 + Q2 =
S = uan ian = Vmax Imax coswt cos(wt − θ)
In a simple series circuit, when ‘Z’ is equal to R + jX
The angle ‘θ’ is positive for current lagging the voltage and
P = |I|2 |Z| cosθ = |I|2R
negative for leading current.
Vmax I max V I Q = |I|2 |Z| sinθ = |I|2X
‘S’ = cos θ (1 + cos 2ωt ) + max max sin θ sin 2 ωt
2 2
The above equation shows that the term containing ‘cosθ’ is  |Z|cosθ = R
V I
always positive and has an average value of P = max max cos θ  |Z|sinθ = X
2
RMS value of current and voltage will give the power expres- The cosine of the phase angle ‘θ’ between the voltage and the
sion P = |V| |I| cosθ; current is called the power factor. An inductive circuit is said to
This average power ‘P’ is also called the real or active power. have a lagging power factor and a capacitive circuit is said to have
Unit for this power is watt. a leading power factor. Power factor indicated whether the current
3.848 | Power Systems

is lagging or leading the applied voltage, the power factor (C) 4.15 kW
is therefore 36.8° 3.12
⎧ ⎛ Q ⎞⎫ kVAR
cosθ = cos ⎨tan −1 ⎜ ⎟ ⎬ or 5.20 kVA
⎩ ⎝ P ⎠⎭
P (D) 3.12 kVAR
cos θ = 36.8°
P +Q 2 2 4.15
kW
Phasor representation of the complex power is given as 5.20 kVA

S = P +Q 2 2
Solution:  (C)
⎛Q⎞ Real power
θ = tan −1 ⎜ ⎟
⎝P⎠
3 VL I L cos θ = 3 × 200 ×15 × cos θ = 4.15 kW

S Reactive power
Q

q 3 VL I L sin θ = 3 × 200 ×15 × sin θ = 3.12 kVAR


P

Figure 1  Power triangle of inductive load Complex power = P 2 + Q 2 = 5.20 kVA

Solved Examples Example 3:  A single-phase load is supplied by a single-


phase voltage source. If the current flowing from the load
Example 1:  A 15 HP, 440 V, three-phase motor is operating to the source is 10 ∠−160oA and if the voltage at the load
at full load, 85% efficiency and 90% power factor lagging, terminals is 100∠60o then the
the real and reactive power drawn by the motor is (A) Load absorbs both real and reactive power
(A) 13,169 W, 14,662 VAR (B) Load delivers both real and reactive power
(B) 13,169 W, 6365 VAR (C) Load absorbs real power and delivers reactive power
(C) 6378 W, 13169 VAR (D) Load delivers real power and absorbs reactive power
(D) 14,662 W, 13,169 VAR
Solution:  (A)
Solution:  (B)
Voltage across load (V) = 100 ∠60°V
15 HP × 746
Current (I ) = = 19.2 A Current entering load (I) = −10 ∠ − 160°V
3 × 440 × 0.85 × 0.9
Real power drawn by the motor =
10 ∠20°
= 3 ×VL ⋅ I L cos θ = 3 × 440 × 19.2 × 0.9 Complex power = VI
=
13,169.12 W = 100∠60o × 10∠20o = 173.6 + j984.8
Reactive power drawn by the motor = 3VL I L sin θ P and Q are positive, so load absorbs both P and Q.

P = 3 × 440 ×19.2 × 0.435


=
6365 VAR Structure of Power Systems
Example 2:  A 25 kVA, 200 V, 50 Hz, 3-f alternator sup- The flow of the electric power from the generating station
plies a line current of 15 A per phase at 0.8 lagging power to the consumer is called an electric power system or elec-
factor and at rated voltage under balance steady-state condi- tric supply systems. It consists of the following important
tion. For this operating condition, the power triangle is components.

(A) 1. Generating station


4.15 kVA
5.20 2. Transmission network
36.8°
kW 3. Distribution network
3.17 kVAR
The figure shows the line diagram of a typical transmission,
(B) 3.12 kVAR distribution scheme and the complete flow of an electric
5.22 power from generation to the consumer premises.
kW
36.8°
4.15 kVA
Chapter 1  Power System Generation, Transmission and Distribution  |  3.849

Generating station service mains to the consumers. This is secondary distribu-


~ 11 KV tion also called low voltage distribution.

Stepup transformer
11 kV/220 kV
Basic Power Generation Concepts
Electricity is not often generated at a power station by elec-
Primary Transmission Stepdown transformer tromechanical generators, primarily driven by heat engines
220 kV/33 kV
fuelled by chemical combustion or nuclear fission but also may
Stepdown transformer
by other means such as kinetic energy of flowing water and
Secondary Transmission
33 KV/6.6 kV wind. The types of electric generation concepts are as follows:

To large bulk loads like 1. Thermal power plants


Primary Distribution factories and industries 2. Hydroelectric power plants
3. Nuclear power plants
4. Diesel power plants
Secondary Distribution
Distribution on transformers 5. Wind plants
6.6 kV/400 V or 230 V 6. Solar power generation, etc.

Thermal Station
Figure 2  Line diagram of typical transmission and
distribution on system
Definition
A generating station which converts heat energy of coal
At the generating stations, an electric power is generated combustion into electrical energy is known as steam station
with the help of three-phase alternators running in parallel. (thermal station).
In the scheme shown in figure, the voltage level is 11 kV but
the voltage level can be 6.6 kV, 22 kV or 33 kV depending Advantages
upon the capacity of the generating station. After generat- 1. Less initial cost.
ing station, actual transmission and distribution starts, the 2. It requires less space as compared to hydroelectric
overall scheme can be divided into four sections which are, power station.
3. Cost of generation is less than that of diesel power
Primary Transmission station.
It is basically with the help of overhead transmission lines.
For the economic aspects, the voltage level is increased to Disadvantages
132 kV, 22 kV or more, with the help of step-up transform- 1. It pollutes the atmosphere due to production of large
ers. Hence, this is also called high voltage transmission. amount of smoke and fumes.
2. Its running cost is more compared to hydro electric
Secondary Transmission plant.
The primary transmission line continuous transmission
Points to decide the selection of site for thermal power
towers till the receiving stations. At the receiving station,
­station
the voltage level is reduced to 22 kV or 33 kV using the
1. Large quantity of cooling water should be available
step-down transformers. This is secondary transmission; the
for the condensers, etc.
conductors used for the secondary transmission are called
2. The fuel to be used in the power station should be
feeders.
cheap and available nearby or ample facilities for its
transportation to the site should be available.
Primary Distribution 3. The soil should be such that, it should provide good
At substations the voltage level is reduced to 6.6 kV, 3.3 kV and firm foundation to the building and the plants.
or 11 kV with the help of step-down transformers. It uses 4. The site should not be surrounded by residential
three-phase three wire underground systems. And the power buildings.
is further transmitted to the local distribution. For the large 5. The future expansion of power station should be
consumers like factories and industries, the power is directly ­possible.
transmitted to such loads from a substation. 6. Ample accommodation for the operational and main-
tenance staff should be available nearby at reasonable
Secondary Distribution rates.
At the local centres, there are step down distribution 7. Facility for the disposal of ash, etc. should be
transformers. The voltage levels of 6.6 kV, 11 kV is fur- ­available.
ther reduced to 400 V using distribution transformers and 8. The cost of land should be reasonable.
3.850 | Power Systems

Exhaust
through(ESP)
Coal
storage

C
h
Coal i
handling m
FD Fan
n
Air Air e
Ash Air heater y
Ash storage Ash Boiled and
disposal handling economizer

Feed Water Flue


Boiler HP ID Fan
gas
pump heaters

Super heat
LP Heaters
steam
Condensate
extraction
pump
Condensate
Steam
Condenser

Turbine
Circulating
water pump

Cooling
towers
~ Generator

Schematic Arrangement of Steam Power Superheater: The steam produced in the boiler is wet
2.
Station and is passed through a superheater where it is super-
heated (i.e. steam temperature increased above that of
1. Coal and ash handling arrangement
boiling point of water).
2. Steam-generating plant
•• Advantage is that efficiency is increased
3. Steam turbine
Economizer: An economizer is a feed water heater,
3.
4. Alternator
i.e. increases temperature of feed water from extract-
5. Feed water
ing heat of flue gases.
6. Cooling arrangement
Air Preheater: It increases temperature of the air
4.
supplied for coal burning by delivering heat from flue
Coal and Ash Handling Plant
gases.
The coal is transported to the power station by road or rail
and is stored in a coal storage plant. Storage of coal is pri- Advantages
marily a matter of protection against coal strikes, failure of •• Thermal efficiency increases.
transportation system and general coal shortages. •• Steam capacity per square metre of boiler surface is
From coal storage plant, coal is delivered to the coal han- increased.
dling plant where it is pulverized. The pulverized coal is fed
to the boiler by conveyor belt. The coal is burnt in the boiler Steam Turbine
and the ash produced after complete combustion of coal is The dry and superheated steam from the superheater is fed
moved to the ash handling plant and then delivered to the to the steam turbine and it converts into mechanical energy.
ash storage plant for disposal. After giving heat energy to the turbine, the steam is exhausted
to the condenser, which condenses exhausted steam.
Steam-generating Plant
Boiler: The heat of combustion of coal in the boiler is
1. Alternator
utilized to convert water into steam at high tempera- The alternator converts mechanical energy into electrical
ture and pressure. energy. The electrical energy is delivered to busbars.
Chapter 1  Power System Generation, Transmission and Distribution  |  3.851

Feed Water cooling water and condensate is the same when leaving
The condensate from the condenser is used as feed water to the condenser.
the boiler. The feed water on its way to the boiler is heated Advantages
by water heaters and economizer. This helps in raising the Low initial cost, less floor area required, less cooling water
overall efficiency of the plant. required and low maintenance charges.

Cooling Arrangement Disadvantages


Condensate is wasted and high power is required for pump-
Water is circulated through the condenser, which takes up
ing water.
heat of the exhausted steam. In case availability of water
•• In a surface condenser, there is no direct contact between
throughout the year is not assured, cooling tower is used.
cooling water and exhausted steam. It consists of a bank
of horizontal tubes enclosed in a cast iron shell. The cool-
Types of Boilers ing water flows through the tubes and exhausted steam
1. Water tube boiler flows over the surface of the tubes.
2. Fire tube boiler
Advantages
•• In water tube boiler, water flows through the tubes Condensate can be used as feed water, less pumping power
and the hot gases of combustion flow over these tubes. required and creation of better vacuum at turbine exhaust.
Advantages are less space requirement, high pressure and
Disadvantages
less liable to explosion, etc.
High initial cost, large floor area requirement and high
•• In fire tube boiler, the hot products of combustion pass
maintenance charges.
through the tubes surrounded by water. Disadvantage is
low pressure.
Types of Steam Turbines
Types of Superheaters 1. Impulse turbine
1. Radiant superheater 2. Reaction turbine
2. Convection superheater •• In an impulse turbine, the steam expands completely in
•• In Radiant superheater, it is placed in the furnace between the stationary nozzles (or flexi blades), the pressure over
water walls and receives heat from the burning fuel the moving blades remaining constant. In doing so, the
through radiation process. Disadvantage is that it may steam attains a high velocity and impinges against the
get over heated and temperature of superheater falls with moving blades.
increase in steam output. •• In reaction turbine, the steam is partially expanded in the
•• In convection superheater, it is placed in the boiler tube stationary nozzles; the remaining expansion takes place
bank and receives heat from flue gases entirely through during its flow over the moving blades.
the convection process.
Advantage is temperature of superheater increases with Efficiency of Steam Power Station
increase in steam output. Thermal Efficiency
The ratio of heat equivalent to the mechanical energy trans-
Types of Air-pre Heaters mitted to the turbine shaft, to the heat of combustion of coal
1. Recuperative type is known as thermal efficiency.
2. Regenerative type
Heat equivalent to the mechanical energy
•• In recuperative type, air-heater consists of a group of steel transmited to the turbine shaft
tubes. The flue gases are passed through the tubes while ηThermal =
air flows externally to the tubes. Heat of coal combustion
•• In regenerative type, air-heater consists of slowly moving •• The thermal efficiency of a modern steam power station
drum. The flue gases flow continuously on one side of the is about 30%
drum and air on the other side.
Overall Efficiency
Types of Condensers
The ratio of heat equivalent of the electrical output to the
1. Jet Condenser
heat of combustion of coal is known as overall efficiency.
2. Surface condenser
Heat equivalent of the electrical output
•• In a jet condenser, cooling water and exhausted steam ηoverall =
are mixed together. Therefore, the temperature of Heat of combustion of coal
3.852 | Power Systems

•• The overall efficiency of steam power station is about 7. The site selected for hydroelectric plant should be
29% ­accessible by rail and road so that necessary equip-
•• Overall efficiency = Thermal efficiency × Electrical ment and machinery could be easily transported.
efficiency
Components of Hydroelectric Plant
Hydroelectric Power Station Dam: A dam is a barrier which stores water and
1.
­creates water head.
Definition
2. Spillways: It discharges the surplus water to down-
A generating station which utilizes the potential energy of stream side of the dam.
water at a high level for the generation of electrical energy 3. Surge tank: A surge tank is located near the begin-
is known as a hydroelectric power station. ning of the conduit. It overcomes the abnormal pres-
Advantages sure in the conduit when load on the turbine falls.
1. It is quiet and clean. It acts as a reservoir during increase of load on the
2. It requires less maintenance. ­turbine.
3. No fuel requirement. 4. Penstocks: Penstocks are open or closed conduits
4. It can be put into service instantly. which carry water to the turbines.
5. It has longer life. •• Low-pressure penstocks
6. In addition to the generation of electrical energy, they E.g.: Cement canals
also help in irrigation and controlling floods. •• High pressure penstocks
E.g.: Steel pipes
Disadvantages 5. Trash rock: It is used to avoid the entry of debris into
1. It involves high capital cost. the pen stock.
2. Availability of huge amount of water. 6. Forebay: It is a small pondage where extra water is
3. Skilled and experienced hands are required to build stored in it and regulates the load variations.
the plant. 7. Reservoir: The area to store water.
4. Increased cost of transmission lines because of re- 8. Turbines: It converts energy of falling water into me-
mote location of plants. chanical energy.
5. Long dry season may affect the delivery of power. 9. Alternator: It converts mechanical energy into elec-
trical energy.
Head works Surge chamber
Spill way
Valve house Types of Turbines
1. Impulse turbine
Penstock It has high power output and low efficiency.
Reservoir Tunnel E.g.: Pelton wheel
Power house 2. Reaction turbines
It has low power output and high efficiency turbines.
E.g.: Kaplan, Francis and Propeller turbines.

Trial race pond Types of Hydro Power Stations


Figure 3  A typical layout of a Hydro plant •• Based on the water availability
1. Run off river plants
(a)  Without pondage
Selection of Site for Hydroelectric (b)  With pondage
Station •• Based on the head available
1. Sufficient quantity of water at reasonable level should 1. Low head (<30 m)
be available. 2. Medium head (30–50 m)
2. The site should allow for strong foundation with min- 3. High head (>50 m)
imum cost. •• Based on the nature of load
3. No possibility of future sources of leakage of water. (1) Base load plant
4. Reservoir should have large catchment area. (2) Peak load plant
5. Local supplies of sand, gravel, etc., should be avail-
able. MW Peak load
6. The land for the construction of the plant should be
available at reasonable price. Base load
Chapter 1  Power System Generation, Transmission and Distribution  |  3.853

Type of Alternator used for Hydroelectric Mass Curve


Power Station A mass curve is a graph drawn between cumulative volume
Air cooled salient pole construction, vertical configuration, of water versus time.
large diameter and small axial length alternator is used. •• A mass curve gives the storage requirement.
Types of turbines used for hydroelectric power station
based on head Nuclear Power Station
High head: Pelton wheel turbines
1. Definition
Medium head: Francis, propeller turbines
2. A generating station in which nuclear energy is converted
Low head: Kaplan turbines.
3. into electrical energy is known as nuclear power station.
Power Advantages
0.736 QWhη 1. Fuel requirement is quite small.
P= kW
75 2. It requires less space.
where
3. Running charges are low.
Q = Discharge in m3/s
4. It can be located near the load centres because it does
W = Water density in 1000 kg/m3
not require large quantities of water and it need not be
h = Height of water in metre
near coal mines.
h = Efficiency
5. Such plants can ensure continued supply because of
Specific Speed large deposits of fuel available.
6. Reliability operation.
With available head of 1 m to develop 1 HP power output,
the speed at which the turbine will rotate is called specific Coolant
speed. Control rods Heat
exchanger Steam
N P
Ns = 5
Turbine

where H 4

P = Power in HP Fuel rods


Generator
N = Actual speed in rpm
H = Head of water Reactor
1.145 N P Water intake
Ns = 5
Moderator

H4
if P is in kW. Disadvantages
1. Fuel is expensive and difficult to recover.
Hydrograph 2. Capital cost is very high.
It is a graph between discharge of water in m3 to time. 3. Radioactive pollution problems.
4. It requires greater technical staff.
5. Not suited for varying loads.
Water Selection of site for nuclear power station
in m3
1. Plant should be located where ample quantity of
Time ­water is available.
•• It gives the variation of water discharge from time to time. 2. Plant should have adequate arrangement for disposal
•• The area under this graph gives the total quantity of water of radioactive waste.
available at a particular period. 3. It should be quite distant from populated areas as
there is a danger of presence of radioactivity in the
Flow Duration Curve atmosphere near the plant.
It is drawn from hydrograph. 4. Transportation facilities should be available.

Schematic Arrangement of Nuclear


m3/sec
Power Station
1. Nuclear reactor
2. Heat exchanger
20% 60% 5% 3. Steam turbine
It is drawn in descending order of water discharge. 4. Alternator
3.854 | Power Systems

Components of Nuclear Reactor Alternator


1. Reactor core: It consists of fuel rods. Alternator is a device which converts mechanical energy
2. Moderator into electrical energy.
•• The purpose of moderator is to slow down the fast Multiplication factor (K)
neutrons.
No. of neutrons released in fission
•• It should have low molecular weight. K=
•• Materials that can be used as moderator are No. of neutrons releasedd in preceding fission
Beryllium, Graphite, H2O and Heavy water (D2O). •• The value of K can be controlled by control rods.
3. Control rods: The purpose of control rods is to absorb •• K > 1, for increasing generation
the fast neutrons and they control the value of K (mul- K = 1, for constant generation
tiplication factor). K < 1, for decreasing generation
•• Materials that can be used for control rods are cad- K = 0, for shut-down generation
mium, boron and Haffinium.
4. Reflector: The purpose of this reflector is to bounce Fission Material
back escaping neutrons into reactor core.
It is the material used as fuel in nuclear power station.
Materials: Reactor grade graphite
5. Shielding: The purpose of shielding is to avoid a, b, g 1. Natural Uranium
radiations outside the reactor core. 99.3% U238
Materials: Stainless steel, etc. 0.7% U235
6. Coolant: The purpose of coolant is to absorb the heat 2. Enriched Uranium
from the reactor core and exchange with water in heat 3. Thorium
exchanger. Materials are 4. Plutonium
Liquid coolants → H2O, D2O
•• ‘Enriched Uranium’ is used in Indian nuclear power
Liquid metals → Na and K
stations
Gases → CO2, H2
•• Fission materials can be divided into two types.
Heat Exchanger 1. Fissile materials
2. Fertile materials
The coolant gives up heat to the heat exchanger which is
utilized in rising the steam. After giving up heat, the coolant •• Fissile materials are those with which chain reactions
is again fed to the reactor. start directly.
Ex: U235, U233 and Pu239
Steam Turbine •• Fertile materials are those with which chain reactions
start indirectly.
The steam produced in the heat exchanger is led to the Ex: U238, Th232
steam turbine through a valve. After doing a useful work
in the turbine, the steam is exhausted to the condenser. The
Critical Mass
condenser condenses the steam which is fed to the heat
exchanger through feed water pump. It is the mass to be added in the chain reaction to maintain
the ‘K’ value at ‘1’ only.
Comparison of Various Power Plants
S. No. Item Steam Power station Hydroelectric power plant Diesel power plant Nuclear power plant
Located at where reservoirs
Located at water and Located away from
1. Site can be obtained by construct- Located at any place
coal available places thickly populated areas
ing dam Example: hill areas
Lower than hydro and Less compared to
2. Initial cost Very high Highest
nuclear plants other plants
Higher than hydro and Except hydro, it has
3. Running cost Very less Highest
nuclear plants minimum running cost
More efficient than
4. Efficiency Least about 25% About 85% About 35%
steam power station
Maintenance
5. Quite high Quite low Less Very high
cost
Transmission
6. Quite low Quite high Least Quite low
and distribution
7. Starting Requires lot of time Started instantly Started quickly Started easily
Chapter 1  Power System Generation, Transmission and Distribution  |  3.855

Economic Aspects
kW
Commonly used terms in system operation are discussed in
the following sections.

Connected Load Time


It is the sum of the continuous ratings of all load-consuming
•• The curve will have only negative slope.
apparatus connected to the system.
•• The area under this curve gives the no. of units generated.
Maximum Demand Integrated Load Duration Curve
It is the highest load demand on the power system during a
It is a graph drawn between MW versus MWh.
given period (day, month, and year).

MW3
Firm Power
MW2
It is the power intended to be always available.
MW1
Cold Reserve
It is the reserve generating capacity available for service but
is not in operation. MWh1 MWh2 MWh3
•• It gives the cumulative value of energy when load is
Hot Reserve changing.
It is the reserve generating capacity in operation but not in •• By using integrated load curve, we can estimate the num-
service. ber of units generated when the load is changing from
MW1 to MW2.
Spinning Reserve
It is the generating capacity which is connected to the bus Load Factor (LF)
and is ready to take load. It is the ratio of average demand to maximum demand
Average demand
Factors Affecting the Cost of Generation Load factor =
Maximum demand
Load Curve
•• Load factor is less than 1.
The curve showing the variation of load on power station •• Higher is the load factor, lesser will be the cost per unit.
with reference to time is known as a load curve.
Demand Factor (DF)
It is the ratio of maximum demand to connected load
kW
Maximum demand
Demand factor =
Connected load

Time
•• It is usually less than 1.
•• Low demand factor results in less capital cost.
•• The variation of quantity w.r.t time, it is known as ‘chron-
ological curve’ Diversity Factor (Div.F)
•• Load curve can have –ve or +ve slopes. Sum of individual maximum demands
•• The area under the curve gives the total number of units Diversity factor =
Maximum demand on power sttation
generated.
•• It is usually more than 1 only.
Total no. of units generated in a period •• Div.F↑, Max. Demand ↓, Installed capacity ↓ cost ↓
•• Average demand =
No. of hours in that period
Load Duration Curve Utilization Factor (UF)
It is a graph drawn from load curve by rearranging load ele- Maximum Demand
U.F =
ments in descending order w.r.t their duration of occurrence. Installed capacity
3.856 | Power Systems

•• It is always ≤ 1. 700
•• If UF is high, i.e. maximum demand is high, then the =
= 1166.6 MW
0.5
power station is having proper utilization.
•• Reserve capacity = Installed capacity – Maximum Reserve capacity = Plant capacity – Maximum demand
demand. = 1166.6 – 1000 = 166.6 MW @ 167 MW
•• If reserve capacity is zero, utilization factor is one. Example 6:  Which of the following stations has minimum
running cost?
Capacity Factor (CF) (A) Hydroelectric (B) Nuclear
Average demand (C) Diesel (D) Coal
Capacity factor =
Installed capacity Solution:  (A)
Actual energy produced No fuel cost
 =
Max energy that could have been produuced
Example 7:  For a hydroelectric power plant with an avail-
•• CF = LF × UF able head of 60 m, which of the following turbines is most
likely to be used?
Plant use Factor (A) Pelton wheel (B) Francis
Station output in kWh (C) Kaplan (D) None of the above
Plant use factor =
Plant capacity × Hours of use
Solution:  (A)
•• Plant use factor is more than or equal to plant capacity
factor. Pelton wheels are typically used for high heads.

Example 4:  A generating station has a connected load of


550 MW. The maximum demand is 380 MW. The number
of units generated per annum is 20 × 108. The demand factor
Power System representation
and load factor are Single Line Diagram
(A) 0.69 and 0.60 (B) 0.60 and 0.69
(C) 1.45 and 0.60 (D) 0.69 and 1.66 Generator or Motor
(Rotating Machine) ∼
Solution:  (A)
Maximum demand Circuit breaker
Demand factor =
Connected load (oil or liquid)

380
= = 0.69 Air Circuit breaker
550
Average demand
Two winding power
No. of units generated per annum transformer
= kW
8760
20 × 108
= = 228.31 MW Three-winding
8760 power transformer
Average demand
Load factor =
   Maximum demand
Current transformer
228.31
= = 0.60
380
Fuse or
Example 5:  A generating station has a maximum demand
of 1000 MW. The annual load factor is 70% and plant
­capacity factor is 60%. The reserve capacity is 3-phase delta
(A) 700 MW (B) 1.166 MW connection
(C) 167 MW (D) 1000 MW
Solution:  (C) 3-phase star connection
Average demand = 0.7 × 1000 = 700 MW (neutral ungrounded)
Average demand Figure 4  (Continued  )
Plant capacity =
Capacity factor 3-phase star connection
(neutral grounded)

Disconnecting Switch
Fuse or

3-phase delta
connection

3-phase star connection


(neutral ungrounded) Chapter 1  Power System Generation, Transmission and Distribution  |  3.857

Above Figure 4 shows the single line diagram of an


3-phase star connection
(neutral grounded) electrical power system. Two generators grounded through
reactors are connected to a bus and through a step-up trans-
former to a transmission line. Another generator, grounded
through reactor, is connected to a bus and through a trans-
Disconnecting Switch former to the opposite end of the transmission line. A load
is connected at each bus.
Reactor
Impedance and Reactance Diagrams
The impedance diagram on single-phase basis for use under
Lighting arrestor balanced operating conditions can be easily drawn from the
online diagram. For the system of Figure 4, the impedance
diagram is shown in Figure 5.
Auto transformer No currents flow in the ground under balanced condi-
tions and the neutral of the generators are at the potential of
Figure 4  Symbols for representing single line diagram the neutral of the system, so the impedance diagram does
not include the current limiting impedances shown in the
Almost all modern power systems are three-phase systems one line diagram between the neutral of the generator and
with the phases of equal magnitude and equal phase dif- ground.
ference (i.e. 120o). These three-phase balanced systems are Since the shunt current of a transformer is usually insig-
always solved as a single-phase circuit composed of one nificant compared with the full load current, the shunt
of the three lines and neutral return and this is sufficient admittance is usually omitted in the equivalent circuit of the
to give a complete analysis. Such a simplified diagram of transformer.
an electric system is called a ‘one-line diagram’ or ‘single- The inductive reactance of a system is much larger
line diagram’. Combined with a standard set of symbols than its resistance. So, the resistance is neglected in fault
for electric components, such one-line diagrams provide a calculations.
compact way to represent information. The purpose of the Synchronous motor loads are always included in making
one-line diagram is to supply in concise form, the signifi- fault calculations, since their generated emfs contribute to
cant information about the system, the importance of dif- the short-circuit current. Induction motors are represented
ferent features of a system varies with the problem under by a generated emf in series with an inductive reactance if
consideration, and the amount of information included on the diagram is to be used to determine the current immedi-
the diagram depends on the purpose for which the diagram ately after the occurrence of a fault. Induction motors are
is intended. Figure 3 below shows the symbols for repre- ignored in computing the current a few cycles after the fault
senting the components of a three-phase power system. occurrence because the current contributed by an induction

1
3
TA TB

Load B
Load A

Figure 5  Single-line diagram of an electrical power system

E1 + +
– E2 –
+
– E
3

Generators Load Transformer Transmission Transformer Load Generator


1 and 2 A TA Line TB B 3.

Figure 6  The per-phase impedance diagram


3.858 | Power Systems

motor dies out very quickly after the induction motor is Single Phase
short circuited. Per-phase reactance diagram after neglect- Assuming that the independent base values are power and
ing all static loads, resistances, shunt admittances of each voltage,
transformer and the capacitance of the transmission line Base volt amperes = Sbase = 1 p.u.
is shown in Figure 6. The per-phase impedance and reac- Base voltage = Vbase = 1 p.u.
tance diagrams are sometimes called the per-phase positive Base active power = Pbase = Sbase.cosf
sequence diagram. Base reactive power = Qbase = Sbase.sinf
XTA XL XTB (VA)base Sbase
Base current = I base = = =1 p.u.
Vbase Vbase
X1 X2 X3 Vbase V 2 base
Base impedance = Zbase = =
I base Sbase
E1 +

+ E

+ E3

2
1
Base admittance = Ybase =
Figure 7  Simplified reactance diagram
Z base
If the actual impedance is Z (ohms), its per-unit value is
given by
Per Unit System
Z Z ( ohms ) × (VA)base
The quantities in a power system (i.e., voltage, current, volt Z( p.u.) = =
amperes and impedance) are often expressed as a per cent or Z base V 2 base
per unit of a base or reference value specified for each. The
Z ( ohms ) × Sbase
per-unit value of any quantity is defined as the ratio of the =
actual quantity to its base quantity. V 2 base
The actual value in any units
Per Unit Value = Three Phase
The base or reference value in the same units
Power and voltage are specified in the same way as single-
The ratio in per cent is 100 times the value in per unit. phase system. But the difference is that the power is speci-
Both the per cent and per-unit methods of calculation are fied as total power (not per phase), and voltage is line-to-line
similar and more informative than the use of actual quan- voltage.
tities. The per-unit method has advantages over per cent In three-phase systems the relations
calculation method because the product of two quantities
expressed in per unit is expressed in per unit itself, but the Pbase = Sbase ⋅ cosf and Qbase = Sbase sinf holds good.
product of two quantities expressed in per cent must be The apparent power S equals
divided by 100 to obtain the result in per cent.
There are several reasons for using a per-unit system. Sbase = 3 Vbase Ibase

1. Similar apparatus (generators, transformers, lines) Sbase


I base = =1 p.u.
will have similar per-unit impedances and losses 3 Vbase
­expressed on their own ratings, regardless of their
Vbase
­absolute size. 2
2. Use of the constant 3 is reduced in three-phase phase V base
Z base = = =1 p.u.
­calculations. I base Sbase
3. Per-unit quantities are the same on either side of a
1 Sbase
transformer, independent of voltage level. Ybase = = =1 p.u.
4. By normalizing quantities to a common base, the Z base V 2 base
­calculations are simplified. If the actual impedance is Z (ohms), its per-unit value is
A per unit system provides units for power, voltage, current given by
and impedance. Only two of these are independent, usually Z Z ( ohms ) × Sbase
Z p.u. = =
power and voltage. Generally base values of power and volt- Z base V 2 base
age are chosen. Once the base power and the base voltage
are chosen, the base current and the impedances are deter- If the impedance has to be represented in a new base value
mined by the natural laws of electrical circuits. denoted as Zbase new (Referred to Sbase new and Vbase new)
The relationship between quantities in a per-unit sys- 2

tem depends on whether the system is single phase or three ⎛ Vbase old ⎞ Sbase-new
Z p.u.new = Z p.u.old × ⎜ ⎟ ×
phase. ⎝ Vbase-new ⎠ Sbase-old
Chapter 1  Power System Generation, Transmission and Distribution  |  3.859

Note: Base impedance with given base values


Zp.u. ∝ Sbase V 2 base
Z base =
1 Sbase
Zp.u. ∝
V 2 base
222 ×106
Example: For Section I Z base1 = = 4.84 Ω
Single line diagram of a power system is shown in the figure. 100 ×106
The system contains one generator, four transformers and 220 2 ×106
two transmission lines. The system ratings and reactances For Section II Z base2 = = 484 Ω
100 ×106
are indicated in the figure. Draw the equivalent impedance
diagram for the given system and per unit equivalent dia- 110 2 × 106
For Section III Z base3 = = 121 Ω
gram with Sbase = 100 MVA and Vbase = 22 kV. 100 × 106

1 T1 3 4 T2 2 112 × 106
Line 1 For Section IV Z base4 = = 1.21 Ω
100 × 106
220kV
Gen 66. 5 MVA Z actual
~ Motor 10.45 kV Per unit impedance calculations Z p.u. =
6 T
Z base
T3 5 Xp.u. = 0.185
90 MVA Line 1 4
2
22 kV Sbase new ⎛ Vbase old ⎞
Xp.u. = 0.18 110 kV 3ph load 10.45kV Z p.u. new = Z p.u.old × ×⎜ ⎟⎟
Absorbs 57 MVA at Sbase old ⎜⎝ Vbase new ⎠
0.6pf (lag)
T1: 50 MVA, 22/220 kV. Xp.u. = 0.1, For generator 1, new per unit reactance
2
T2: 40 MVA, 220/11 kV. Xp.u. = 0.06, 100 ⎛ 22 ⎞
X sg = X sg, old × × = 0.2 p.u.
T3: 40 MVA, 22/110 kV. Xp.u. = 0.064, 90 ⎜⎝ 22 ⎟⎠
T4: 40 MVA, 110/11 kV. Xp.u. = 0.08,
Line 1 : 48.4 Ω, Line 2 : 65.43 Ω Xt 1 = j 0.2 Xtl1 = j 0.1 Xt 2 = j 0.15

Reactance Diagram Xt 3 = j 0.16 Xtl 2 = j 0.5 Xt 4 = j 0.2


Xsg = j 0.2 Xsm = j 0.25

1 3 4 2
Xt 1 Xl 1 Xt 2 Eg ~ 0.95 ~ Em
j1.266

5 6 Figure 8  Per-unit impedance diagram


Xsg Xt 3 Xl 2 Xt 4 Xsm

Rl 48.4
Eg ~ ~ Em For Transmission line 1, X l p.u. = = 0.1 p.u.
484
Xl
65.43
For Transmission line 2, X lp.u. = = 0.5 p.u.
121
Per Unit Representation
2
100 ⎛ 22 ⎞
Vbase2 = 220 kV For Transformer 1, X t1 = 0.1 × × ⎜ ⎟ = 0.2 p.u.
Sec. 1 Sec. 2 Sec. 4 50 ⎝ 22 ⎠
Sec. 1 4 For Transformer 2,
1 3 2
2
⎛ 100 ⎞ ⎛ 22 ⎞
X t2 = 0.06 ⎜ = 0.15 p.u.
⎝ 40 ⎟⎠ ⎜⎝ 22 ⎟⎠
Gen
~ Motor For transformer 3,
5 6 2
⎛ 100 ⎞ ⎛ 220 ⎞
X t3 = 0.064 ⎜ = 0.16 p.u.
⎝ 40 ⎟⎠ ⎜⎝ 220 ⎟⎠
For transformer 4,
Sec. 3 2
⎛ 100 ⎞ ⎛ 220 ⎞
X t4 = 0.08 ⎜ = 0.2 p.u.
⎝ 40 ⎟⎠ ⎜⎝ 220 ⎟⎠
Vbase 1 = 22 kV Vbase 3 = 110 kV Vbase 4 = 11 kV
3.860 | Power Systems

2 Example 11: A single-phase system is shown in figure


For motor, X Sm = 0.185 ⎛⎜
100 ⎞ ⎛ 10.45 ⎞
⎟⎜ ⎟ = 0.25 p.u. below:
⎝ .5 ⎠ ⎝ 11 ⎠
66
For three-phase load:
Power factor: cos–1(0.6) = 53.13o 500 Ω
Load S3f = 57 ∠53.13 A B C

V 2 rated 10.452
Z act = = =1.1495 + j1.5326 Ω
S* 57∠ − 53.13
13.8/138 kV 13.8/69 kV
1.1495 + j1.532 15 MVA 15 MVA
Per unit impedance of 3-f load = x = 10% x = 8%
1.21
With the base in ‘A’ circuit chosen as 13.8 kV and
= 0.95 + j1.2667 p.u.
15 MVA, the impedance diagram is

Example 8: The base impedance and base voltage of a (A) j 0.1 j 0.08
345 kV system are chosen to be 3000 A and 300 kV, respec-
tively, the base impedance of the system is
(A) 115 Ω (B) 100 Ω 500
(C) 10 Ω (D) 0.01 Ω

Solution:  (B)
300 ×103 Vb (B) j 0.1 j 0.08
Base impedance = =
3000 Ib
= 100 Ω
0.634
Example 9: Let a 10 kVA, 400/200-V transformer be ap-
proximately represented by a 4 Ω reactance referred to the
low-voltage side, considering the rated values as base quanti-
ties, what is the transformer reactance as a per unit quantity. (C) j 0.08 j 0.08
(A) 1 p.u. (B) 0.5 p.u.
(C) 0.25 p.u. (D) 1.5 p.u.
1.57
Solution:  (A)
V2 200 2
Base impedance (ZB) = = =4 Ω
SB 10, 000
(D) j 0.1 j 0.08
The per unit reactance referred to the low-voltage side is
4
Per unit reactance = = 1 p.u.
4 1.57

Example 10:  The per-unit impedance of an alternator cor-


responding to base values 13.2 kV and 25 MVA is 0.18 p.u.
The per unit value of the impedance for base values are 13.8 Solution:  (D)
kV and 60 MVA in p.u. will be
(A) 0.395 p.u. Chosen base values 13.8 kV and 15 MVA
(B) 0.472 p.u. Transformer 1 Xp.u. = 0.1 p.u. [old and new base values
(C) 0.329 p.u. are same]
(D) 0.082 p.u. Transformer 2 Xp.u. = 0.08 p.u. [old and new values are
same]
Solution:  (A)
S 15 ×106
2 Load Zp.u. = Zactual × 2base = 500 × =1.57 p.u.
⎛ Vbase new ⎞ ⎛ VAbase old ⎞ ( )
2
V base 69 ×103
Z p.u.new = Z p.u.old ⎜ ⎟⎟ × ⎜⎜ ⎟⎟
⎜V
⎝ base old ⎠ ⎝ VAbase new ⎠
Example 12: The zero sequence reactance in p.u. are
2
⎛ 60 ⎞ ⎛ 13.2 ⎞ ­indicated in the network shown in the given below. The zero
= 0.18 × ⎜ ⎟ ⎜ = 0.395 p.u.
⎝ 25 ⎠ ⎝ 13.8 ⎟⎠ sequence driving point reactance of node-3 will be

Chapter 1  Power System Generation, Transmission and Distribution  |  3.861

1 XT0 = 0.12 p.u. 2 3 4

XT0 = 0.12 p.u.


X0 = 0.08 p.u.

Xg0 = 0.2 p.u.


Xg0 = 0.1 p.u.

(A) j0.2 p.u. (B) j0.32 p.u. Conductor Materials


(C) j0.123 p.u. (D) j0.52 p.u. The conductor is one of the important items as most of the
Solution:  (C) capital outlay is invested for it. The proper choice of mate-
rial and size of the conductor is of considerable importance.
Reactance diagram is given by The conductors should have the following properties
1 2 3 4 1. High electrical conductivity
j 0.1 j0.12 j 0.08 j 0.12 j 0.2
2. High tensile strength
3. Low cost
4. Low specific gravity
All above properties are not found in single material.
Driving point reactance at node 3 = j(0.12 + 0.08)||j(0.12 So, combination of different materials will give above
properties.
0.2 × 0.32
+ 0.2) = j = 0.123 p.u. The common materials used for overhead lines are cop-
0.52 per, aluminium steel-cored, aluminium galvanized steel
and cadmium copper. Conductors are preferably stranded
Mechanical Design of in order to maintain flexibility. In stranded conductors one
central wire around this some no of conductors are used
Transmission Lines based on the voltage designed according to the 6, 12, 18,
Introduction 24…..wires.
Electric power can be transmitted or distributed by means
of overhead lines and underground cables, the underground
cables are used rarely because of two main reasons. They steel
are (i) high initial cost (ii) high Insulation cost. Hence, the
Al
power is transmitted by overhead lines by proper mechani-
cal factors like strength weather conditions and other exter- If there are ‘n’ layers, then, 3n(n + 1) +1 is total no of
nal interferences taken into consideration because of the individual layers. In the manufacture of stranded conduc-
continuity of operations in the line. tors, the layers of wires are twisted in opposite directions so
Components required for overhead lines that layers are bounded together.
The successful operation of an overhead line depends to
a great extent upon the mechanical design of the line. Copper
The components of the OH lines are Copper is an ideal material for OH lines due to its high con-
1. Conductors ductivity and tensile strength.
2. Poles •• High electrical conductivity
3. Insulators •• Greater tensile strength
Lighting
•• High current density
arrestor •• High cost
Insulators •• Non-availability
Conductors Specific gravity of copper is 8.9 gm/cc. Due to disadvantage
Arms of copper, they are rarely used.
Danger
plates Aluminium
Pole
•• Aluminium is cheap and light in weight compared to
copper.
3.862 | Power Systems

•• Smaller conductivity and low tensile strength. 3. Longer life


•• The conductivity of aluminium is 60% that of copper. 4. Light in weight
•• The diameter of Aℓ conductor is 1.26 times the diameter
Different types of poles
of copper for equal resistances.
1. Wooden poles
•• The increased cross sections of Aℓ give greater surface to
2. Steel poles and steel towers
wind pressure, so supporting towards should be in greater
3. RCC (Reinforced concrete) poles
in strength.
•• Specific gravity of Aℓ is 2.71 gm/cc.
Insulators
Galvanized Steel The overhead line conductors are supported on the poles in
•• Galvanized steel conductors can be used for extremely such a way that the current should not flow to earth by poles,
long spans. so insulators are properly fixed to conductor to provide nec-
•• They are suitable in rural areas where cheapness is the essary insulation between line conductors and support.
main consideration. The Insulators should have following properties
•• It has poor conductivity due to high resistance of steel. 1. High mechanical strength
Cadmium Copper 2. High relative permittivity
3. High electrical
Conductor material in this case is copper alloyed with
cadmium. Addition of 2% cadmium to copper increases Different types of Insulators
50% tensile strength and 15% reduction of conductivity. 1. Pin type
Therefore, these conductors can be useful for exceptionally 2. Suspension type
long spans. 3. Strain type
4. Shackle type
ACSR (Aluminium Conductor Steel Reinforced)
•• The aluminium conductor is reinforced with a core of gal- Pin-type Insulator
vanized steel wires. •• The pin type is secured (fixed) to cross arm on the pole.
•• The diameter of both steel and Aluminium is same. •• Pin-type insulators are used for transmission and distribu-
•• The cross section of two metals is generally in the ratio tion of electrical power at voltage up to 33 kV.
of 1:6.
Groove for conductor
•• They are represented by x/y.
•• Steel-cored aluminium spans can be used for towers of Sheds
smaller heights.
•• Steel-cored aluminium conductor will produce smaller
Sag, and hence longer spans can be used with smaller
tower heights.
Cementing
Example 13:  An ACSR conductor consists of 30 Aℓ and
7 steel stands Galvanized
30 Steel pin
Solution:  Ans.
7  
Example 14:  ACSR conductors represented by 7/54 Causes of Insulator Failure
•• Insulators are required to withstand both mechanical and
Solution:  7 steel and 54 Al electrical stress.
•• The electrical breakdown of the insulator can occur either
Diameter of Stranded Conductor by flash – over or puncture.
D = (2n − 1)d Flashover: an arc occurs between the live conductor and
 d = diameter of each strand insulator pin.
n = no of layers Puncture: the discharge occurs from the conductor to pin
through the body of the insulator.
Poles Puncture strength
The supporting structure for overhead lines are poles and Safety factor of insulator =
Flash − Overvoltage
towers.
Mechanical–Electrical
The poles or towers should have following properties
1. Flashover: Arc occurs between line conductor and insu-
1. High mechanical strength lator or pin (i.e. Earth) and the discharge jumps over
2. Cheap in cost air gap, following the shortest path. After flashover,
Chapter 1  Power System Generation, Transmission and Distribution  |  3.863

insulator continues to work properly, unless destroyed Tower


by the heat produced during flashover.
2. Puncture: Discharge focus from pin to the body of V1
insulator. This destroys the insulator and it must be Metal link
replaced.
V V2

Suspension-type Insulator
•• Suspension-type insulator consists of a number of porce- V3
lain discs connected in series by metal links in the form
of a string. The conductor is suspended at the bottom end
of this string while the other end of the string is secured Conductor
to the cross arm of the tower. Figure 10  A typical suspension-type Insulator
•• Each disc is designed for low voltage 11 kV. For example,
6 discs → 66 kV. C
•• These insulators are cheaper than pin-type insulators for C Ic1 I1 1 V1

voltages beyond 33 kV. C2 V2


•• We can replace the damaged disc from the whole string C Ic2 I2
V
and we can add the desired number of discs.
C3 V3
•• These insulators are used with steel towers. C Ic3 I
3
I4
C4 V4

Figure 11  Potential distribution across a string of Insulators

Porcelain portion of each disc between two metallic links


forms a capacitance. This capacitance is known as self-
capacitance. If there were mutual capacitance alone, the
potential distribution across each unit would have been the
same. But in actual case, capacitance also exists between
metal fitting and tower or earth. This capacitance is known
as shunt capacitance. Due to the shunt capacitance, charg-
ing current is not the same through all the discs of the string.
Therefore, voltage across each disc will be different.
Figure 9  Suspension-type insulators
Presence of self-capacitance and shunt capacitance results
in the
Strain Insulators 1. Non-uniform potential distribution of voltage across
When there is a dead end of the line or there is corner or the individual discs of suspension-type insulators.
sharp curve, the line is subjected to greater tension. 2. The disc nearest to the conductor has maximum volt-
Strain insulator is used for low voltages line (<11 kV). age across it.
3. The disc nearest to the conductor is under maximum
•• By assembling suspension insulators, we can use as the electrical stress and is likely to be punctured.
strain insulator. 4. Since the insulator capacitances are ineffective for
Strain DC, the voltage across each disc would be same for
DC voltage across the string.
The unequal potential distribution is generally expressed in
insulator Conductor terms of string efficiency:

Pole
String Efficiency
The ratio of voltage across the whole string to the product of
discs and the voltage across the disc nearest to the conduc-
Potential Distribution Over a String of tor is known as ‘string efficiency’.
Insulators String efficiency
A string of suspension insulators consist of a number of Voltage across the string
=
porcelain discs connected in series through metallic links. n × Voltage across disc nearest to conductor

3.864 | Power Systems

n = no. of discs in the string. the bottom unit and connected to the metal work at the bot-
Capacitance per insulator KC tom of this unit and therefore to the line. The guard ring
Let K = = introduces capacitance between the metal fittings and the
Capacitance to ground C
line conductor. The guard ring is connected in such a way
where C1 = C2 = C3 = C4 = KC that shunt capacitance currents ic1 , ic2 , ic3, etc. are equal to
Let V be the operating voltage (Line to ground) and V1, V2, metal fitting line Capacitance currents im1 , im2 , im3, etc. The
V3 and V4 the voltage drops across the units as shown in the result is that same charging current ‘I’ flows through each
figure. unit of string. Consequently, there will be uniform potential
V = V1 + V2 + V3 + V4. distribution across the units.

From the figure, we can write kc


C Ic I1
I2 = I1 + Ic1 1 im 1

kc
wkcV2 = wkcV1 + wcV1 ( w = supply frequency) C I
c2
I2 i
m2
⎛1+ K ⎞ ⎛ 1⎞
V2 = V1 ⎜ = V1 ⎜1 + ⎟ . Guard ring
⎝ K ⎟⎠ ⎝ K⎠ I3

⎛ 3 1 ⎞
V3 = V1 ⎜1 + + 2 ⎟
⎝ K K ⎠
Figure 12  Static shielding
⎛ 6 5 1 ⎞
V4 = V1 ⎜1 + + 2 + 3 ⎟ . Common Data for Examples
⎝ K K K ⎠
A 3-f overhead transmission line is being supported by
V1 can be expressed in terms of V and from this V2, V3, etc. three disc insulators. The potential drop across top unit and
can be obtained normally. middle unit are 10 kV and 12 kV, respectively.
If k = 5, then V2 = 1.2 V1, V3 = 1.64 V1, V4 = 2.40 V1
Example 15: The ratio of capacitance between pin and
\ V1 < V2 < V3 < V4. earth to self-capacitance of each unit is
As ‘k’ increases, the division of voltage becomes uniform. (A) 0.2 (B) 5
The 100% string efficiency can be achieved by increasing (C) 0.833 (D) 1.2
‘K’ value till the potential distribution across discs become Solution:  (A)
uniform.
The equivalent circuit of string insulators is shown below
Methods of Improving String Efficiency I1
C V1
1. Selection of K KC i1
2. Grading of units (discs) A
I2 V2
3. Static shielding KC i2 C V
B
I3 V3
Selection of K C
KC i3
Performance of string (string efficiency) depends on the C
value of ‘K’. As K is increased the division of voltage
becomes more equalized. To increase the ‘K’ value, longer
cross arm for the tower to be used.
Conductor location
Grading of Units Let ‘K’ be the ratio of capacitance between pin and earth to
Insulators of different dimensions are chosen so that each self-capacitance.
disc has different capacitance. The insulators are capaci- ‘C’ is self-capacitance of each unit.
tance graded such a way that the top unit has the minimum Capacitance between pin to earth = KC
capacitance and the capacitance increases progressively as Apply KCL at node A
the bottom unit is reached. This method has a disadvantage I2 = I1 + i1
that a large number of different sized insulators are required. V2 wc = V1 wc + V1 k wc
V2 = V1(1 + k)
Static Shielding
V2 −V1 12 −10
The string efficiency in this method is improved by provid- k= = = 0.2
ing grading in the form of a large metal ring surrounding V1 10
Chapter 1  Power System Generation, Transmission and Distribution  |  3.865

Example 16:  The string efficiency of the system is The difference in level between points of supports and the
(A) 10.6% (B) 12% lowest point on the conductor is called sag.
(C) 78% (D) 92%
A   B
Solution:  (C) 2 2

From the given data,


P
V1 = 10 kV, V2 = 12 kV x_ S
2
V3 = V2 + (V1 + V2) k y
= 12 + (10 + 12) × 0.2 = 16.4 kV
T
Voltage between line and earth = V1 + V2 + V3 O wx
= 38.4 kV
X
Voltage across string
String efficiency = ×100 Consider a conductor between two equilateral supports A
No.of insulators×V3
and B with ‘O’ as the lowest point as shown above.
38.4
= × 100 = 78% Wl 2
3 × 16.4 Sag S =
8T
Example 17:  For the following figure, the voltage across where l = Length of span
middle insulator as a percentage of line voltage to earth W = Weight per unit length of conductor
T = Tension in the conductor
0.1 C i C 0.25 C Let A and B be at different levels form the ground.
V1 a I1
A i1 B
V2 0.1 C I C 0.25 C h
2 A
B i2 X1
V3 ib
i3 S2
S1

(A) 29.85% (B) 40.65%


(C) 70.15% (D) 59.35% O X2

Solution:  (A) l

Applying Kirchhoff’s law at node ‘A’ Wx12 l Th


S1 = where x1 = −
I2 + ia = I1 + i1 2T 2 Wl
V2wc + 0.1 wc(V2 + V3) = V1 wc + 0.25V1 wc Wx2 2 l Th
V2 + 0.1V2 + 0.1V3 = 1.25V1 S2 = where x2 = +
2T 2 Wl
1.1V2 + 0.1V3 = 1.25V1
where h = Difference in levels between two supports.
At node ‘B’ I3 + ib = I2 + i2
Wl
0.25V2 + 1.25V2 = 1.1V3 S2 – S1 = [ x2 − x1 ] .
2T
From the above two equations, we have
V1 = 0.988 V2 and l = x2 + x1
V3 = 1.362 V2
Effect of Wind and Ice
V = V1 + V2 + V3
V = 0.988 V2 + V2 + 1.362 V2 = 3.35 V2 The effect of ice covering (wi) is to increase the weight of
the conductor (wc) and thus increases vertical sag. To guard
1 against this, the tension on the conductors at the time of
V2 = V = 029.85 V
3.35 erection is suitably decreased.
V2 = 29.85% of V If the wind pressure is also considered, the wind pres-
sure acting on the projected area of the conductor per metre
Sag Calculation in OH Lines converted to local should be added to the weight. If Ww is
While erecting an overhead line, it is very important that wind pressure.
conductors are under safe tension. In order to permit safe Resultant weight (per conductor)
tension in the conductors, they are not fully stretched but
W= (Wc + Wi ) + Ww 2
are allowed to have a dip or sag.
3.866 | Power Systems

Ww
12 × 3975
q l= = 135 m
2.621
W The span length = 2 × 135 = 270 m.
(Wc + Wi)

Example 18:  If the ultimate strength is 5750 kg, sag is 2 m Electrical Design of Overhead
and factor of safety is 2 and the overhead line has a span of
250 m. The weight of the conductor is Lines
(A) 0.736 kg (B) 184 kg An AC transmission has resistance inductance and capaci-
(C) 92 kg (D) 23 kg tance uniformly distributed along its length. These four are
called parameters. These parameters will determine the effi-
Solution:  (B)
ciency of transmission system.
Allowable maximum tension Parameters of Transmission line:
Ultimate strength 5750 1. Resistance (R)
(T) = = = 2875 kg
Safety factor 2 2. Inductance (L)
3. Capacitance (C)
Total sag(s) = 2 m 4. Conductance (G)
R L
WL2 ~
S=
8T
8 ST 2 × 8 × 2875 C G
W= =
L2 ( 250 )
2

Conductance
Neglected
W = 0.737 kg/m
w2 l2 The transmission line consists of a series combination of
Half-span length = l + resistance, inductance and a parallel combination of capaci-
6T 2
2
tance, conductance. Conductance is normally neglected due
(0.737)2 ⎛⎜⎝
250 ⎞ to leakage over line insulation is almost always in overhead

250 2 ⎠ transmission line.
= + = 125 m
6 × ( 2875) The series line parameters, i.e., inductance resistance are
2
2
uniformly distributed along the line and they together form
Total length = 250 m series impedance. Inductance is the most dominant line
Weight of conductor = (0.737 × 250) kg parameter for a power system. It is the inductive reactance
= 184 kg which limits the transmission capacity of line.
Example 19:  An overhead line conductor is subjected to a
horizontal wind load of 1.78 kg/m and vertical ice loading Resistance
of 1.08 kg/m. If the maximum possible sag is 6 m. Safety It opposes the flow of current in a line conductor. It is the
factor is 2. Weight of conductor is 0.844 kg/m2. The tension cause for power loss in the Transmission line
is 7950 kg. The permissible span between two supports is
(A) 135 m (B) 275 m
(C) 13 m (D) 27 m ρl
R=
a
Solution:  (B) R = Resistance
( wc + wi ) ρ = Resistivity
2
Total weight w = + ww 2
l = Length of conductor
a = Area of conductor
= (0.844 + 1.08)2 + (1.78)2 The variation of resistance of metallic conductor with tem-
= 2.621 kg/m perature is practically linear.
wl 2 R2 = R1 [(1 + a1(t2 − t1)]
Maximum possible sag D =
2T α0
a1 =
2.621 × l 2
1 + α 0 t1
6=
2 × (7950 2) a0 = temperature coefficient
Chapter 1  Power System Generation, Transmission and Distribution  |  3.867

Skin Effect From Equations (2) and (3), we obtain


An alternating current flowing through the conductor does yI
Hy = AT/m (4)
not distribute uniformly. Rather it has the tendency to con- 2π r 2
centrate near the surface of the conductor. This effect is The flux density (By) at ‘y’ metre from the centre of conduc-
known as skin effect. tor is
The skin effect depends on μ yI
By = mHy = Wb/m 2 (5)
1. Nature of material 2π r 2
2. Diameter of wire where m is the permeability of the conductor.
3. Frequency Consider now an infinitesimal tabular element of thick-
4. shape of wire ness dy and length 1 metre. The flux in the tabular element
When frequency < 50 Hz and conductor diameters < 1 cm, y y 2
(df = Bydy) weber links the fractional turn = resulting
skin effect can be negligible.  r2
in flux linkages of
Flux Linkages ⎛ y2 ⎞ ⎛ y 2 ⎞ μ yI
dl = ⎜ 2 ⎟ dϕ = ⎜ 2 ⎟ dy Weber-turns (6)
⎝ r ⎠ 2π r
2
Inductance of a circuit is defined as the flux linkages per ⎝r ⎠
unit current. Integrating, we get the total internal flux linkages as
There are two important cases of flux linkages.
μI μI
r
Flux linkages due to a single current carrying conductor
The magnetic lines of force will exist inside the conduc-
lint = ∫ 2π r
0
4
y 3 dy =

Wb-T/m

tor and outside of the conductor. Contribution of both fluxes For relative permeability mr = 1 (non-magnetic conductor)
will form the inductance
m = 4p × 10–7 Wb-T/m
For example and
A long straight cylindrical conductor of radius ‘r’ metre and 1
carrying a current of ‘I’ Amperes Lint = × 10 −7 H/m
2
Flux
Flux Linkage Due to Flux Between Two
r
I r Points External to Conductor
Figure shows two points P1 and P2, at distance D1 and D2
from a conductor which carries current of I amperes. The
y magnetic field external to the conductor is concentric cir-
dy
cles around the conductor and therefore all the flux between
P1 and P2 lies within the concentric cylindrical surfaces
passing through P1 and P2.
Internal Flux
Flux
Figure shows the cross-sectional view of a long cylindrical
conductor carrying current I D1
P1 dy
The mmf around a concentric closed circular path of radius P1
(internal) of the conductor is
[ Hy = magnetic field intensity] (AT/m)
D2
∫ Hy ds = Iy (i) (Amperes law), where P2

Iy = current enclosed (A) Magnetic field intensity at distance y from the conductor is
By symmetry, Hy is constant and is in direction of ds all 1
along the circular path. Therefore from equation (1) Hy = AT/m
2π y
2p y Hy = Iy(2) The flux df contained in the tabular element of thickness
dy is
Assuming uniform current density μI
df = dy Wb
⎛ π y2 ⎞ ⎛ y2 ⎞ 2π y
I y = ⎜ 2 ⎟ I = ⎜ 2 ⎟ I (3) The flux df links all the current in the conductor once and
⎝ πr ⎠ ⎝r ⎠
only once.
3.868 | Power Systems

Flux linkages, y = flux linkages


μI dϕ di di
dl = 1 × df = dy e = ⋅ V =L V
2π y di dt dt
Total flux linkages of the conductor due to flux between dϕ
points where L = is defined as the inductance of circuit in
di
D2 henry
μI μ D2
l12 = ∫ 2π y dy = 2π ln D Wb-turns/m
L = Henry
ϕ
D1 1
i
where ‘ln’ stands for natural logarithm. y = Li (Wb-T)
Since mr =1 m = 4p × 10–7 If the current is alternative
D2 l = LI
l12 = 2 × 10–7 ln Wb-turns/m Inductance of a single-phase two wire line
D1
The inductance of the conductor by the flux included I1 I2
between points P1 and P2 is then r1 r2
D2 D – r2
L12 = 2 × 10–7 ln H/m
D1
Or D
D
L12 = 0.461 log 2 mH/km D + r2
D1
The flux linkage caused by current in Conductor 1 is given
Flux linkages due to External flux: by
Let the external point be at distance D from the centre of D
the conductor, due to external flux, these linkages can be is l1 = 2 × 10–7 ln
r′
obtained by substituting D1 = r and D2 = D, i.e. Inductance of the conductor due to current in conductor 1
D only is
lext = 2 × 10–7 ln D
r L1 = 2 × 10–7 ln
r1′
Total flux linkages of the conductor due to internal and
external flux are Due to current in conductor 2 only is
D
l = lint + lent L2 = 2 × 10–7 ln
r2′
I D
= × 10 −7 + 2 × 10 −7 ln D
2 r L = L1 + L2 = 4 × 10–7 ln H/m
r1′r2′
⎛1 D⎞ D
If r1 ′ = r2 ′ = r′ ; then
2 × 10–7 ⎜ + ln ⎟ = 2 × 10–7 ln −1/ 4
=
⎝4 r⎠ re
L = 4 × 10–7 ln D/r′ H/m
D
=
2 × 10–7 ln L = 0.921 ln D/r′mH/m.
r′

where r′ = re–1/4 = 0.77888 r


Inductance of Composite Conductor Live

D
l = 2 × 10–7 ln Wb − T/m
r′
Inductance of the conductor due to flux up to an external
point Ι⎛ 1 1 1 1 ⎞
⎛D⎞  li = 2 × 10–7 ⎜ ln + ln + ....... + ln .....ln ⎟
L = 2 × 10–7 ln ⎜ ⎟ H/m ⎜
n ⎝ Di1 Di2 Dii Din ⎟⎠
⎝ r′ ⎠
I ⎛ 1 1 1 ⎞
Inductance (L) =
2 × 10–7 ⎜ ln + ln + .....in ⎟

m1 ⎝ Di1 Di2 Dim1 ⎟
•• It opposes sudden changes in currents. The voltage ⎠
induced can be obtained by
(D )
1/ m ′
i1 ’ .....Di2 ’.....Din ’
dϕ =
2 × 10 –7
I ln Wb-T/m
e= V
(D D )
1/ n
dt i1 Di2 Dil ’ in
Chapter 1  Power System Generation, Transmission and Distribution  |  3.869

The inductance of filament is then Inductance of Three-phase Lines

Li =
λi
= 2n × 10 −7 Ln
( Dil ’.....Dij ’.....Dim’ ) H/m The basic equations developed can however be easily
adapted to the calculation of the inductance of three-phase

I /n (
Di1 Di2 ....Dii ......Din ) lines.
Figure shows conductor of a three-phase live
L1 + l2 + l3 + ....Ln
Lavg = Unsymmetrical spacing:
n b
 Conductor A is composed of n filaments electrically in
parallel
D12
Lavg L1 + l2 + .....Ln
LA = = D23
n n2 a

LA = 2 × 10 −7 D31
1/ m′n

⎡( D1i ′ ....D1 j ′ ....D1m′ ) ... ( Dil ′ ...D1 j ′ ..Dim′ ) ... ( Dnl ’...Dnj ′ ...Dnm ′ ) ⎤ c
Ln ⎣ ⎦
Practically transposition is difficult but mathematical it can
( ) ( ) ( )
1/ n2

⎡ D ′ ....D ....Dn′ ... D ′ ...D ′ ...Dn′ ... D ′ ...D ...Dn ⎤ be assumed.


⎢⎣ 1i 1j il 1j nl nj ⎥⎦
⎛ GMD ⎞
L/phase = 2 × 10–7 ln ⎜ ⎟ H/M
⎛ Productsof all possible mutualinductances ⎞ ⎝ GMR ⎠
LA = 2 × 10 −7 ln × ⎜ ⎟
⎝ Geometric mean radius ⎠ ⎛ DM ⎞
L/phase = 2 × 10–7 ln ⎜ ⎟ H/M
DM DM ⎝ DSA ⎠
LA = 2 × 10–7 Ln H/m = 0.461 Long mH/km
DSA DSA
GMD or DM = 3 XYZ if D12 = X, D23 = Y, D31 = Z
Total inductance of line
 GMR or DSA = r′
L = LA + LB Symmetrical lines
X = Y = Z = d (equilateral spacing)
Problems on Self-GMD ⎛ GMD ⎞
L/phase = 0.2 Ln ⎜ ⎟ mH/km
⎝ GMR ⎠
Example 20:  A conductor is composed of seven (7) iden- GMD = d
tical copper strands each having a radius r, as shown in GMR = r′
­Figure 1. Find the self-GMD of the conductor. Inductive reactance
D14 = 4r X = 2pFL
D12 = 2r
D26 = 2 3 r
Unsymmetrical Spacing
4 When three-phase line conductors are not equidistant from
5 3 each other, the conductor spacing is said to be unsymmetri-
7
6 2 cal. So, flux linkages are different. Interchanging the posi-
1 tions of the conductor at regular intervals along the line
such an exchange of positions is known as transposition.
Solution:  The self-GMD of seven strand conductor is the c b
a
49th root of the 49 distances. Thus,
c
b a

( ) ( 2r ) )
6 1/ 49
DS = ((r′)7 D 2
12 D 2
26 D14 D17 6
c b a

Ia + Ib + Ic = 0
Substituting the value of various distances, Ia = I (1 + j0)

( (
DS = ( 0.7788r ) 22 r 2 × 3 × 22 r 2 × 22 r 2 × 22 r × 2r × 2r
7
) )
6 1/ 49 Ib = I (−0.5 – j0.866)
Ic = I (−0.5 + j0.866)
⎛ 1 1 1 ⎞
2r ( 3 ( 0.7788 ) ) la1 = 2 × 10–7 ⎜ I a ln + I b ln + I c ln
1/ 7
 ⎟ Wb-T/m
DS = ⎝ ra′ D12 D31 ⎠
61/ 49
⎛ 1 1 1 ⎞
DS = 2.177r  la2 = 2 × 10–7 ⎜ I a ln + I b ln + I c ln ⎟ Wb-T/m
⎝ ra′ D23 D12 ⎠
3.870 | Power Systems

⎛ 1 1 1 ⎞ Section 1 of transposition cycle


la3 = 2 × 10–7 b ⎜ I a ln + I b ln + I c ln ⎟ Wb-T/m
⎝ ra′ D31 D23 ⎠ (2 Dh)
=
12

Average flux linkages of conductor ‘a’ are


Deq = 21/ 6 D1/ 2 p1/ 3 h1/ 6
λa1 + λa 2 + λa3
la =
Dsa = ( r ′q r ′q ) = ( r ′q )
1/ 4 1/ 2
3
⎛ 1 1
Dsb = ( r ′h r ′h ) = ( r ′h )
1/ 4 1/ 2
= 2 × 10 −7 ⎜⎜ I a ln + I b ln
( D12 D23 D31 )
1/ 3
ra

Dsc = ( r ′q r ′q ) = ( r ′q )
1/ 4 1/ 2
1 ⎞
+ I c ln 1/ 3 ⎟
( D12 D23 D31 ) ⎟⎠ Equivalent self-GMD Ds = ( Dsa Dsb Dsc )
1/ 3

Ib + Ic = −Ia ( r ′)1/ 2 q1/ 3 h1/ 6


=
⎡ ⎛ p⎞ ⎤
1/ 2 1/ 3
( D12 D23 D31 ) L = 2 × 10–7 ln ⎢ 21/ 6 ⎛⎜ ⎞⎟ ⎜ ⎟ ⎥ H/phrase/m.
1/ 3
D
l0 = 2 × 10 Ia ln –7
ra′ ⎢⎣ ⎝ r ′ ⎠ ⎝ q ⎠ ⎥⎦
Mutual inductance between the two circuits
Deq = ( D12 D23 D31 ) = equivalent equilateral spacing
1/ 3
2/3
⎛ p⎞
M = 2 × 10–7 ln ⎜ ⎟
⎝q⎠
Deq Deq
La = 2 × 10–7 ln = 2 × 10 −7 ln H/m M → 0. \ mutual impedance is zero.
ra′ DS
3 2D
Dm = Deq L = 1 × 10–7 ln
r′
ra = rb = rc
Capacitance
La = Lb = Lc
The capacitance and conductance together forms the shunt
where Ds = self-GMD, Dm = Mutual GMD admittance of a transmission line. When an alternating volt-
age is applied to the line, the line capacitance draws a lead-
Double-circuit Three-phase Line ing sinusoidal current called the charging current which is
The double-circuit three-phase line is built so as to increase drawn even when the line is open circuited at the far end.
transmission reliability, at enhanced cost. +Q
The three sections of the transposition cycle of two par- –Q
r
allel circuit three-phase line with vertical spacing are shown B
A
below.
a c b d
h c′ h b′ h a′
p p p ∞ ∞
b a c Q −Q
b′ a′ c′ VA = ∫ dx + ∫ dx
q q q r
2π xε 0 d
2π xε 0
c b a
a′ c′ b′ Q ⎛ ∞ ∞⎞
Section - 1 Section - 2 Section - 3
= ⎜ log e r − log e d ⎟ V
2πε 0 ⎝ ⎠
Deq = ( Dab Dbc Dca )
1/ 3
Q d
= log e V
Dab = mutual GMD between phases a and b in 2πε 0 r
Section 1 of transposition cycle Potential difference between conductor B and central
‘infinite’
= ( DP DP ) = ( DP )
1/ 4 1/ 2


−Q Q

Dbc = mutual GMD between phases b and c in VB = ∫r 2π xε 0 dx + ∫d 2π xε 0 dx
Section 1 of transposition cycle

( Dp )
1/ 2
= −Q ⎡ d⎤
= ⎢log e r ⎥ V
Dca = mutual GMD between phases a and c in 2πε 0 ⎣ ⎦
Chapter 1  Power System Generation, Transmission and Distribution  |  3.871

−Q d Assume balanced supply, then


= log e V
2πε 0 r QA + QB + QC = 0
2Q d QB + QC = −QA
VAB = 2VA = log e V
2πε 0 r 1 ⎛ 1 1 ⎞ QA d
VA = ⎜ QA log e r − QA log e d ⎟ = 2πε log e r V
2πε 0 ⎝ ⎠
Q 0
CAB = Q/VAB = F/m
2Q d Capacitance of conductor A with respect to neutral,
log e
2πε 0 r QA QA 2πε 0
CA = = F/M = F/m
πε 0 VA Q d d
CAB = F/m A
log e log e
d 2πε 0 r r
log e
r Unsymmetrical Spacing
Capacitance to Neutral Three-phase transposed line having unsymmetrical spacing
The aim is to know the capacitance between one of the con- for balanced condition QA + QB + QC = 0
ductors and a neutral point between them. Since potential of Capacitance from conductor to neutral is
the midpoint between the conductors is zero, the potential QA 2πε 0
difference between each conductor and ground or neutral is CA = = F/m
VA 3 d1d2 d3
half the potential differences between the conductors. log e
r
A B
Example 21:  A single-phase transmission line has two par-
N allel conductor 4 m apart, radius of each conductor being
CAN CBN 2 cm. calculate the capacitance of line per km .given that
Capacitance of neutral e0 = 8.854 × 10–12 F/m.

CN = CAN = CBN = 2CAB Solution:  Conductor radius = 2 cm


Spacing of conductors = 4 m = 400 cm
2πε 0 Capacitance of the line
CN = F/m.
d πε 0
log e = F/m
r d
Capacitance of a Three-phase log e
r
The capacitance of each conductor is considered instead of π × 8.85 × 10 −12
capacitance from conductor to conductor = F/m
400
Two cases: log e
2
1. Symmetrical spacing = 5.247 × 10–12 F/m
2. Unsymmetrical spacing = 5.247 × 10–3 m F/km

Symmetrical Spacing Example 22:  A three-phase overhead transmission line has


its conductors arranged at the corners of an equilateral tri-
The three conductors A, B and C are having charges of QA, angle of 8 m side. Calculate the capacitance of each line
QB and QC per metre (equidistant of d metre) from each conductor per km. diameter of each conductor is 2.5 cm.
other, the capacitance from line conductor to central in the
symmetrical spaced line. Solution:  Conductor radius r = 2.5/2 = 1.25 cm
A Spacing of conductors d = 8 m = 800 cm
Capacitance of each line conductor
d d
2πε 0 2 × π × 8.854 × 10 −12
= F/m = F/m
d ln
d ⎛ 800 ⎞
B C ln ⎜ ⎟
r ⎝ 1.25 ⎠
∞ ∞ ∞
QA QB QC  = 8.609703 × 10–12 F/m
VA = ∫r 2π xε 0 ∫d 2π X ε 0 ∫d 2π X ε 0 dx
dx + +
   = 8.6097×10–3 m F/km
1 ⎛ 1 1 1⎞
= QA log e + QB log e + QC log e ⎟ Example 23:  Calculate the capacitance of a 400 km long
2πε 0 ⎜⎝ r d d⎠
three-phase 50 Hz overhead transmission line consisting of
1 ⎡ 1 1⎤ 3 conductors of each diameter 2 cm and spaced 4 m at the
= QA log e + ( QB + QC ) log e ⎥
2πε 0 ⎢⎣ r d⎦ corner of an equilateral triangle.
3.872 | Power Systems

Solution:  Equilateral spacing d = 4 m = 400 cm a


R = 2/2 = 1 cm
Capacitance of each conductor to neutral
2πε 0 2πε 0 2π × 8.854 × 10 −12 1.5 m 1.5 m
= F/m = F/m
d log e 400 /1 log e 400 /1
log e
r
c 2.5 m b
9.28 × 10–9 F/km = 9.28 × 10–3 mF/km
=
(A) 1.36 mH (B) 136 mH
Example 24: A long two-wire line composed of solid (C) 6.817 mH (D) 681 mH
ground conductors is 0.5 cm and the distance between their
centres is 2 m. If this distance is double, then the inductance Solution:  (B)
per unit length. GMD
Inductance per phase = 2 × 10–7. ln
(A) Halves GMR
(B) Increases but does not double
GMD = 3 Dab Dbc Dca = 3 1.5 × 1.5 × 2.5 = 1.778 m
(C) Doubles
(D) Decreases but does not halve GMR = 0.778 r = 0.778 × 0.25 cm
= 0.1945 × 10–2 m
Solution:  (B) GMD
Inductance per phase = 0.2 ln mH/km
⎛d⎞ GMR
Inductance of conductors per phase = 0.2 ln ⎜ ⎟ mH/km
⎝ r′ ⎠ ⎛ 1.778 ⎞
= 0.2 ln ⎜
⎛ 2 ⎞ ⎝ 0.1945 × 10 −2 ⎟⎠
L1 /phase = 0.2 ln ⎜ ⎟ mH/km
⎝ 0. 778 × 0.5 × 10 −2 ⎠ = 1.36 mH/km

Total inductance per phase = 1.36 × 100 km = 136 mH
After distance is doubled
Example 27:  A 125 km long transmission line is loaded at
⎛ 4 ⎞ 110 kV. If the loss of line is 20 MW and the load is 175 MVA
L2 /phase = 0.2 ln ⎜ −2 ⎟
mH/km
⎝ 0.778 × 0.5 × 10 ⎠ the resistance of the line is
(A) 7.9 Ω/phase (B) 4.56 Ω/phase
L2/phase > L1/phase but L2/phase ≠ 2L1/phase
(C) 13.68 Ω/phase (D) 45.6 Ω/phase
Inductance per unit length has increased but not doubled
Solution:  (A)
Example 25:  The conductor of 15 km long, single-phase,
Given line loss expression is given by
two-wire line is separated by a distance of 1.5 m. The diam-
eter of each conductor is 0.75 cm. If the conductors are of Ploss = I L2 R
copper, the inductance of the circuit is PL 175 × 106
(A) 2.22 mH (B) 22.8 mH IL = = = 1.6 kA
V 110 × 103
(C) 1.10 mH (D) 11.0 mH
( )
2
Ploss = 20 × 106 = I L2 R = 1.6 × 103 R
Solution:  (B)
R = 7.9 Ω/phase
Inductance of 1-f circuit = 2 L per phase
Common Data Questions
d
= 2 × 0.2 ln
r′ Example 28:  A 100 km long, three-phase, 110 kV 50 Hz
Distance (d) = 1.5 m, overhead transmission line has three conductors each diam-
eter 1.5 cm. The conductors are spaced 2 m at the corners of
Radius (r′) = 0.778 × 0.75 × 10–2m
equilateral triangle. The capacitance of the line is
⎛ 1.5 ⎞ (A) 1 × 10–3 mF (B) 2.29 F
L = 0.4 ln ⎜ −2 ⎟
mH/km

⎝ 0.778 × 0.75 × 10 ⎠ (C) 1.5 F (D) 1.5 mF
= 2.2L mH/km
Solution:  (A)
Length of the line is 10 km 2π ∈0
Total inductance = L × 10 km Capacitance of the line (C) =
d
L = 22.2 mH ln
r
2π × 8.854 ×10 −12
Example 26: A 100 km long three-phase transmission = =10 ×10 −12 F/m
lines are shown in the figure. The diameter of each conduc- ⎛ 200 ⎞
ln ⎜ ⎟
tor is 0.5 cm. The value of the inductance per phase is ⎝ 0.75 ⎠
Chapter 1  Power System Generation, Transmission and Distribution  |  3.873

Capacitance of 100 km line = 10 × 100 ×10–12 where VR1 is the receiving end voltage under No-load
C = 1 × 10–9 condition.
VR2 is the receiving end voltage under full load condition.
Example 29:  The capacitance to neutral is
Note that VR1 and VR2 are magnitudes only
(A) 8.4 μF/km (B) 0.0119 μF/km
Regulation is calculated on
(C) 8.4 F/km (D) 0.0084 F/km
1. Speed
Solution:  (B)
2. Voltage
a b c
3m 3m Performance of rotating equipment is evaluated based on
efficiency and speed regulation.
0.242 Performance of static equipment is evaluated based on
Cn = Efficiency and voltage regulation.
Deq
ln Since transmission line comes under static equipment,
r
performance is evaluated when the system reaches steady
Deq = (3 × 3 × 6)1/3 = 3.78 m
state rather than in transient state.
0.0242 •• The loss in a transmission line is due to real power but not
 Cn = = 0.0119 μF/km
3.78 due to reactive power. Hence, the efficiency is calculated
ln
0.5 in terms of real power.
Example 30:  The changing current per kilometre line is
(A) 0.379 A/km (B) 0.167 A/m
Classification of Overhead Transmission
(C) 0.379 A/m (D) 0.23 A/km Lines
•• In steady-state condition, for a line whose lengths is less
Solution:  (D) than 160 km, the voltage or current does not vary much
1 1 and for the line lengths of about 160 km, the parameters
Xn = = can be assumed as lumped but not distributed. These lines
ωCn 2 × π × 50 × 0.0119 × 10 −6
are called as electrically short transmission lines. These
= 267.48 × 103 Ω/km electrically short transmission lines are again divided as
⎛ 100 ⎞ short and medium transmission lines.
Vn ⎜ ⎟ × 1000
Charging current = ⎝ 3⎠ Short Transmission Lines
=
Xn 267.4 × 103 The lines of lengths up to 80 km are known as short trans-
= 0.23 A/km mission lines in which the effect of shunt capacitance is
neglected.
Performance of Lines Medium Transmission Lines
Performance of transmission lines is to find the efficiency
The lines of lengths above 80 km and below 160 km are
and Regulation.
known as medium transmission lines in which the shunt
The efficiency of the transmission line is %
capacitance is assumed to be lumped at the centre (or) half
Power delivered at the receiving end of the capacitance is lumped at each end of the line
Efficiency =  ×100
Power sent from sending end
Long Transmission Lines
Power delivered at the receiving end
×100 For line lengths more than 160 km, all the parameters are
Power sent from sending end
distributed and in some cases where the line length is less
Receiving end in a transmission line is where the load is than 250 km, the analysis is done using nominal-p and T.
connected and the supply is from the sending end.

Regulation Source
R Load
The regulation of the line is defined as the change in the
receiving end voltage, expressed as the percentage of full B
load voltage from no - load to full load, keeping the sending Y
end voltage and frequency constant.
VR1 − VR2
% Regulation = × 100
VR2
3.874 | Power Systems

•• The above figure shows a balanced 3-f source which is The last term in the square root can be neglected.
connected to a balanced star load. As the sum of all the 1/ 2
current in a balanced polyphase network is zero, hence the ⎧⎪ ⎛ 2 I R ⎞ 2I X ⎫⎪
current through the wire connected between the star point Vs = Vr ⎨1 + ⎜ r ⎟ cos ϕr + r sin ϕr ⎬
⎩⎪ ⎝ Vr ⎠ Vr ⎪⎭
of the load and neutral of the system is zero. Hence, the
star point of the load and the neutral point of the source Using Binomial expansion, and limiting only to two terms,
are at the source potential. Hence, for all purposes, a 3-f
balanced system can therefore be analysed on a single- Vs ≈ Vr + Ir R cosfR + IrXsinfR
phase basis. Here, Vs is the sending end voltage corresponding to a par-
ticular load current and power factor condition. From the
Short Transmission Line equivalent circuit of short line, we can see that the receiving
•• The parameters considered here are the lumped param- end voltage under no load Vr′ is same as the sending end
eters. Lumped parameters are those which can be placed voltage under full-load condition, i.e.
at one point (or) two points to the maximum, and these Vr′ = Vs
parameters have physical and electrical separation. These
parameters are used to study the steady-state behaviour of Vs − Vr
\ % Regulation = ×100
the long transmission lines. Vr

•• The inductance will be high and capacitance will be
less in short transmission lines. Hence the capacitance ⎛I R I X ⎞
 = ⎜ r cos ϕr + r sin ϕr ⎟ ×100
is neglected. Therefore the parameters present are only V
⎝ r V ⎠
r

resistance and inductance.
IR I X
•• The equivalent circuit and the vector diagram for a short Per unit Regulation = r cos ϕr + r sin ϕr
transmission line are shown in the below figure. Vr Vr

XL
= ur cosfr + ux sinfr
Is R Ir
where ur and ux are the per unit values of the resistance and
reactance of the line, respectively.
•• The performance of transmission line can be analysed at
Vs Vr a faster rate with the help of less no. of equations by rep-
resenting the transmission line in a 2-port network model
rather R–L–C model.

Vs Two-port Network Model


Port: pair of terminals (phase and Neutral) is known as
Ir x
Vs sin fs ports.
Ir R
Two ports
Vr
Receiving end
fs
fr Sending end
Ir
Vs cos fs I1 I2
A C
V1 Network V2
The vector diagram is drawn taking Ir, the receiving end B D
current, as the reference
From the vector diagram, As we want V1, I1 from V2, I2, we take the transmission line
as A B C D networks.
Vs cosfs = Vr cosfr + Ir R(7)
Is Ir
A C
Vs sinfs = Vr sinfr + Ir X(8)
Vs Vr known
Squaring and adding equations (7) and (8), B D

Vs2 = Vr2 + 2 IrRVr cosfr + 2IrXVrsinfr + Ir2 (R2 + X 2) The unknown sending end electrical quantities are expressed
in terms of known receiving end electrical quantities along
with certain parameters.
2 I r R cos ϕr 2 I r X sin ϕr I r2 2
⇒ Vs = Vr 1 +
Vr
+
Vr
+ 2 R + X2
Vr
( ) Those parameters are called A B C D parameters.
Vs = A Vr + B Ir. kV/phase
Chapter 1  Power System Generation, Transmission and Distribution  |  3.875

Is = C Vr + D Ir kA/phase •• For short transmission lines, regulation is equal to voltage


drop but this is not possible in medium and long transmis-
⎡Vs ⎤ ⎡ A B ⎤ ⎡Vr ⎤
⎢ ⎥=⎢ ⎥⎢ ⎥ sion lines, as shunt capacitance is present.
⎣ I s ⎦ ⎣C D ⎦ ⎣ I r ⎦ •• In case of short transmission line, if load is open, then
If A = D → Symmetrical network |Vr| = |Vs| and Is = 0. But this is not true in case of medium
If A D – B C = 1 → Reciprocal network and long transmission lines as shunt capacitance is pre-
sent and hence Is ≠ 0 but Ir = 0 and Vs = Vc.
(or)
– A D + B C =1 Condition for Maximum Regulation
•• Electrical network may be symmetrical (or) unsymmetri- The expression is given by
cal but always reciprocal network
% e = % Vr cosfr + % Vx sinfr
Units for A, B, C, D The condition for maximum regulation can be obtained
from
Open Circuit at the Receiving End
d %ε
= 0
Is Ir = 0 dϕr
A C
Vs ~ Vr0
⇒ – % Vr sinfr + % Vx cosfr = 0
B D Ir X
× 100
%Vx Vr X
Vs = A Vr ⇒ tanfr = = =
0
%Vr I R R
Is = C Vr
r
× 100
Vr
0

V ⎛X⎞
A = s | Ir = 0 (unitless)
Vr0 ⇒ fr = tan–1 ⎜⎝ R ⎟⎠

Is X – Line reactance Impedance triangle
C = Ir = 0 R – Line resistance
Vr0
‘C’ open circuit admittance in Mho’s
Vr − Vr Z
X
% Regulation = 0 ×100
Vr
q
Impedance triangle R
Vs
− Vr X
A tan q =
%e= ×100 R
Vr
When the phase angle of load is equal to the impedance
angle, the regulation is maximum.
Short Circuit at Receiving End
X
Is Ir = 0 ⇒ tanfR = = tanq
A C R
Vs ~ Vr0 = 0
⇒ fR = q
B D
When the receiving and is short circuited Vr = 0. Then
Condition for Zero Regulation
ABCD parameters are given by % e = % Vr cosfr – % Vx sinfr = 0
Vs = B Ir Ir R
× 100
Is = D Ir %Vr Vr R
⇒ tanfr = =
%Vx I r X X
Vs B × 100
⇒ = Vr
Is D

R ⎛π ⎞
V ⇒ tanfr = = cot θ = tan ⎜ − θ ⎟
B= s x ⎝2 ⎠
Ir
Vr = 0

π
(Short-circuit impedance in ohms) ⇒ fr = − θ
2
Is π
D= Vr = 0 ⇒ fr + q =
Ir 2
3.876 | Power Systems

•• Max regulation occurs for lagging load, 


 

(ii) Angle between Vs and Vr
% e = % Vr cosfr + % Vx sinfr ⎛ 291.6 ⎞
a = tan–1 ⎜ = 0:49°
•• Min regulation occurs for leading load, ⎝ 33, 999.95 ⎟⎠
% e = % Vr cosfr – % Vx sinfr \ Sending power factor angle,
(negative value) fs = fr + a = 36°86′ + 0°49′ = 37°35′
•• We get % regulation as ‘0’ only in the leading loads \ Sending end power factor, cosfs = cos(37° 35′) = 0.79
provided lagging
X (iii) Line losses = I2R = (41.67)2 × 15 = 26.045 kW
(i) = 1.0
R Output = 1100 kW
(ii) 0.707 lead p.f. Power sent = 1100 + 26.045 kW
=1126.045 kW
Example 31: A single-phase overhead transmission line \ Transmission efficiency
delivers 1100 kW at 33 kV at 0.8 p.f. lagging. The total Power delivered
resistance and inductive reactance of the line are 15 W and = × 100
Power sent
20 W, respectively. Find the (i) sending end voltage (ii) send-
ing end power factor and (iii) transmission efficiency. 1100
= × 100 = 97.68%
Solution:  Load power factor, cosfr = 0.8 lag 1126.045
Total line impedance → Z = R + jXL Example 32:  What is the maximum length in km for a single-
= 15 + j20 phase transmission line having copper conductor of 0.775 cm2
Receiving end voltage, VR = 33 kV cross-section over which 200 kW at unity power factor and at
kW × 103 3300 V are to be delivered? The efficiency of transmission is
\ Line current, I =
VR × cos ϕR 90%. Take specific resistance as 1.725 mW cm.

Solution:  Receiving end power = 200 kW
1100 × 10 3
= = 41.66 Transmission efficiency = 0.9
33, 000 × 0.8
2, 00, 000
cosfR = 0.8 \ Sending end power =
0.9
⇒ sinfR = 0.6 =2,22,222 W
\ Line losses = 2,22,222 – 2,00,000
= 22,222 W
200 × 103
Line current I = = 60.6 A
Vs IXL 3300 × 1
Let R W be the resistance of the 2 conductors, hence line
VR
fS losses = 2 I 2R
a
fR 22,222 = 2 I 2R
I
22, 222
 ⇒ R= = 3.025 W
VR = VR + j0 = 33,000 V 2 × (60.6 )
2

I = I(cosfR – j sinfR) ρI R× A
\ R= ⇒l=
= 41.67 (0.8 – j 0.6) A ρ
= 33.33 – j25 = 41.67 ∠−36.86° 3.025 × 0.775
=−6 = 1.36 × 106 cm

    1.725 × 10
(i) Sending end voltage, Vs = VR = + I * Z
=
13.6 km
=
33,000 + (33.33 – j25.0) (15 + j20)
=
33,999.95 + 291.6 j Medium Transmission Lines
We know that the transmission lines with lengths between
( 33, 999.95) + ( 291.6 )
2 2
\ Magnitude of Vs = 80 km and 160 km are categorized as medium length
= 34,001.20 V lines, in which the parameters are assumed to be lumped.
Chapter 1  Power System Generation, Transmission and Distribution  |  3.877

The shunt capacitance is assumed to be either at the middle To calculate the regulation, we have to find Vr′, the receiv-
of the line or half of the total capacitance is concentrated at ing end no load voltage keeping the sending end voltage Vs
each end of the line. These two configurations are classified fixed in magnitude. The above nominal-T network reduces
as—nominal-T and nominal-p, respectively. The circuits to the network beside.
are shown below. ⎛−j⎞
Vs ⎜ ⎟
R X R X
⎝ ωc ⎠
+j +j Vr′ =
2 2 Vc 2 2 ir R x j
R +j −
Is S 2 2 ωc
Ic \ The regulations for the nominal-T network is obtained as
V ′ − Vr
V1 V2
% regulations = r × 100
Y = jwc Vr
The efficiency is obtained as
Power at receiving end
%h= × 100
Power at receiving + Loss
Figure 13  Nominal-T network
In order to determine A, B, C, D constants for nominal-T,
R + jx I Ir ⎛ Z⎞
R Vc = Vr + Ir ⎜ ⎟
Is ⎝ 2⎠
Y
2 Ic = VcY
V1 Y V2
2 ⎛ Z⎞
Is = Ir + Ic = Ir + VcY ⇒ Ir + ⎜Vr + I r ⎟ Y
⎝ 2⎠
⎧ Z ⎫ Z
Vs = Vc + Is Z = Vr + Ir Z + ⎨ I r + ⎛⎜Vr + I r ⎞⎟ Y ⎬
Figure 14  Nominal-p network 2 2 ⎩ ⎝ 2⎠ ⎭ 2

⎛ YZ ⎞ ⎛ Z Z YZ 2 ⎞
Nominal-T Network =
Vr ⎜1 + ⎟ + Ir ⎜ + + ⎟
⎝ 2 ⎠ ⎝2 2 4 ⎠

To analyse the medium length lines by this network, it
would be convenient to take receiving end current as the ⎛ YZ ⎞ ⎛ YZ 2 ⎞
=
Vr ⎜1 + + I ⎜ Z + ⎟
reference vector rather than taking the receiving end voltage 2 ⎟⎠
r
⎝ ⎝ 4 ⎠

(Vr) as the reference. The vector diagram for lagging power
factor load is given below. ⎛ YZ ⎞ ⎛ YZ ⎞
Is = Ir ⎜1 + + VrY = YVr + ⎜1 + 2 ⎟ I r
From the network diagram, in order to calculate the regu- ⎝ 2 ⎟⎠ ⎝ ⎠
lation, we have to first calculate the sending end voltage Vs.
By comparing the above equations with the standard ones,
⎛R x⎞
VC = (|Vr| cosfr + j·|Vr| sinfr) + Ir ⎜ + j ⎟ Vs = AVr + BIr
⎝2 2⎠
Ic = jwc Vc
Is = CVr + DIr
Is = Ic + Ir = Ir + jwc Vc
⎛R x⎞ YZ ⎛ YZ ⎞ ⎛ YZ ⎞
 Vs = Vc + Is ⎜ + j ⎟ ⇒ A=1+ , B = Z ⎜1 + ⎟ , C = Y, D = ⎜1 + ⎟
⎝2 2 ⎝ 4 ⎠ ⎝ 2⎠
2⎠
⎛R x⎞ We can see that A = D,
 (|Vr| cosfr + j |Vr| sinfr) + Ir ⎛ R + j x ⎞ + Is ⎜ + j ⎟
=
⎜⎝ ⎟⎠ ⎝ 2 2 ⎠ 2
2 2 ⎛ YZ ⎞ ⎛ YZ ⎞
and AD – BC = ⎜1 + ⎟ − YZ ⎜1 + ⎟
⎝ 2⎠ ⎝ 4⎠
R/2
Y 2Z 2 Y 2Z 2
=
1+ − YZ − YZ − = 1
4 4
|VS | jX/2

Nominal-p Representation
Vr′ Nominal-p representation of medium length transmission
lines and their phasor diagram are shown below.
3.878 | Power Systems

Is R + jx I1 Ir To find the A, B, C, D constants for nominal-p network,


R
Ic1 Y
Ic 2 Ic = Vr
Y = j wc 2
Y = j wc 2 2 VrY
Vs Vr I1 = Ir + Ic = Ir +
2 2
2
⎛ VY ⎞ ⎛ YZ ⎞
Vs = Vr + I1Z = Vr + ⎜ I r + r ⎟ Z = ⎜1 + Vr + ZI r
⎝ 2 ⎠ ⎝ 2 ⎟⎠
IC1
VsY V Y ⎧ ⎛ YZ ⎞ ⎫ Y
IC2 Is = I1 + Ic2 = I1 + = I r + r + ⎨Vr ⎜1 + ⎟ + ZI r ⎬
2 2 ⎩ ⎝ 2 ⎠ ⎭ 2
Vs
⎛ Y 2 Z ⎞ ⎛ YZ ⎞
= Vr ⎜ Y + ⎟ + 1+ Ir
i1x ⎝ 4 ⎠ ⎜⎝ 2 ⎟⎠
Vr
IC2 Comparing the coefficients of equations with the standard
I1
I1R
ones,
Ir YZ
A = 1 + ,
From the above representation, 2
ωC B = Z
Ic = jVr ωC , I1 = |Ir| (cosfr – jsin fr) + jVr
1
2 2 ⎛ YZ ⎞
C = Y ⎜1 + ⎟
Vs = Vr + I1Z ⎝ 4⎠
⎧ VrωC ⎫ ⎛ YZ ⎞
Vr + ⎨ I r ( cos ϕr − j sin ϕr ) + j
= ⎬ (R + jX) D = ⎜1 + ⎟
⎩ 2 ⎭ ⎝ 2⎠
Vsω C It is seen that A = D,
and Is = I1 + Ic2 = I1 + j
2 ⎛ YZ ⎞
2
⎛ YZ ⎞
and AD – BC = ⎜1 + ⎟ − ZY ⎜1 + ⎟ = 1
jVrω C jVsω C ⎝ 2⎠ ⎝ 4⎠
Is = |Ir| (cosfr – j sinfr) + +
2 2
Example 33:  A three-phase 50 Hz overhead transmission
⎧ jVrωC ⎫
Vs = Vr + ⎨ I r ( cos ϕr − j sin ϕr ) + ⎬ (R + jX)] line 100 km long has the following constants:
⎩ 2 ⎭
Resistance/km/phase = 0.1 W
To calculate the regulation, we have to find no load receiv-
Inductive reactance/km/phase = 0.2 W
ing end voltage by keeping the sending end voltage con-
stant. The nominal-p circuit for no-load reduces as follows. Capacitive susceptance/km/ phase = 0.04 × 10–4 mho
Determine (i) sending end current (ii) sending end
voltage (iii) sending end power factor and (iv) trans-
R mission efficiency when supplying a balanced load of
10,000 kW at 66 kV, p.f. 0.8 lagging, use nominal-T
jX method.
|VS |
Solution:  Total resistance/phase, R = 0.1 × 100
jwc V r′ = 10 W
2
Total resistance/phase, XL = 0.2 × 100
= 20 W
⎛ −2 j ⎞ Capacitive susceptance,
Vs ⎜ ⎟
⎝ ωC ⎠ Y = 0.04 × 10–4 × 100 = 4 × 10–4 W
Vr ′ =
2j Receiving end voltage/phase
R + jX −
ωC
66, 000
V ′ − Vr VR = = 38,105 V
\ % Regulation = r × 100 3
Vr
10, 000 × 103
P Load current, IR = = 109 A
and %h= × 100 3 × 66 × 0.8
P + 3I12 R cosfr = 0.8, ⇒ sinfr = 0.6
Chapter 1  Power System Generation, Transmission and Distribution  |  3.879

Impedance per phase, Z = R + j XL = 10 + j20 fS = q1 + q2 = 6′ + 29°40′ = 29°46′
Taking receiving end voltage as the reference vector, \ Sending end power factor, cosfs
we have,
 = cos 29°46 = 0.868
Receiving voltage,  VR = VR + j0 = 38,105 V
\ Sending end power = 3 VsIs cosfs

Load current, I R = IR (cosfr – jsinfr) = 3 × 40,130 × 100 × 0.868 = 10,449.85 kW
= 109 (0.8 – j0.6) = 87.2 – j65.4 Power delivered = 10,000 kW


   Z 10000
Voltage across C, V1 = VR + I R \ Transmission efficiency = × 100 = 95.7%
2 10, 449.855
=
38,105 + (87.2 – j 65.4) (5 + j 10)
Long Transmission Lines
=
38,105 + 436 + j 872 – j 327 + 654 If the lines are more than 160 km in length, for the analysis
to be accurate, the parameters must be taken as distributed
= 39.195 – j 545 rather than lumped for the voltage and currents.

 

Charging current, I c = jY V1 l + ∆l
Is Ir
= j 4 × 10 (39,195 – j 545)
–4

= 0.218 + j 15.6
Vs V+∆V V Vr
Sending end current

   
Is = I R + Ic

=
(87.2 – j 65.4) + (0.218 + j 15.6) ∆X
X

=
87.4 – j 49.8 = 100 ∠–29°40′ = 100 A
Let Z = Series impedance per unit length
Sending end voltage,
 y = Shunt admittance per unit length

    Z l = Length of the line
Vs = V1 + I s
2 Z = zl = Total series impedance
 = (39,195 – j 545) + (87.4 – j 49.8) (5 + j 10) Y = yl = Total shunt admittance
= 40130 + j 80 For analysis, the elemental length dx is redrawn

\Line-to-line sending end voltage = 40130 × 3 V + ∆V


I + ∆I V,I
 =
69,507 V = 69.507 kV
Z∆X
XL XL
R R
2 2 Vc 2 2 IR
C∆x
Y∆x
Ic

Vs C V1 LOAD
∆x

We take the receiving end as the reference for measuring


distances. Take an elemental length dx of the line at a dis-
VS tance ‘x’ from the receiving end. Let the voltage and current
at a distance ‘x’ are V and I and at a distance x + dx, V + DV
VR
and I + DI, respectively.
fS DV = Iz Dx ⇒ DI = Vy Dx
IS
f1 f2
ΔV ΔI
⇒ Iz ⇒ = Vy
fR
IR Δx Δx
Ir
3.880 | Power Systems

When Dx → 0, the equations become, During high frequencies (or) surges, normally the losses are
neglected and hence, the characteristic impedance becomes
dV
= Iz(9) surge impedance. ‘Surge impedance loading’ of a line is
dx the power transmitted when the line is terminated through
dI a resistance equal to surge impedance. A line terminated in
= Vy(10)
dx its characteristic impedance is called a flat line or an infinite
Differentiating Equation (9),we get line.
The lower value of surge impedance in case of cables is
d 2V dI d 2V
⇒ = Z = Z. y. V ⇒ = Z yV = 0 due to relatively large capacitance and low inductance of
dx 2 dx dx 2 the cables.
The solution of the above equation is The propagations constant g = a + jb, the real part is
known as attenuation constant (a) and the quadrature com-
V = A exp ( yz .x) + B exp (– yz .x)
ponent b is known as the phase constant and is measured in
z radians per unit length.
Let Zc = and g = yz = a + jb V + I r Z c aX jbX
y In the above equation, the first part, r e e
2
where Zc is known as characteristic impedance and ‘g ’ is the
is called the incident voltage wave as its value increases
propagation constant.
as ‘X’ is increased, this means that the magnitude of volt-
The above equations can be rewritten as
age wave decreases in magnitude as it travels towards the
V = A eg X + B e–g X receiving end. Similarly, the second part of the equation
1
I=
Zc
(A eγ X + B e −γ X) Vr − I r Z c −α X − j β X
e e is called reflected voltage. At any point
2
The constants can be determined by the boundary condi- on the line, the voltage wave is the sum of the incidence and
tions, at x = 0, reflected waves. The above result is true even for the current
V = Vr and I = Ir ⇒ Vr = A + B equation.
1 V +I Z V +I Z The above equations for voltage and current can be
Ir = ( A − B ) ⇒ A = r r c and B = r r c re-arranged as follows,
Zc 2 2
Substituting ‘A’ and ‘B’ in above equations eγ X + e −γ X eγ X − e −γ X
V = Vr + I r Zc
Vr + I r Z c γ X Vr − I r Z c −γ X 2 2
V = e + e
2 2 Vr cos h g X + Ir Zc sin h g X
=
1 ⎡Vr + I r Z c γ X Vr − I r Z c −γ X ⎤ 1 ⎡ eγ X + e −γ X eγ X + e −γ X ⎤
and I= e − e ⎥ and I= V + I Z
Z c ⎢⎣ 2 2 ⎦ Zc
⎢ r
⎣ 2
r c
2



We can observe that V and I are the functions of the distance 1
x, receiving end voltage Vr and current Ir and the parameters = [Vr sin γ X + I r Zc cos h γ X ]
Zc
of the line, which means they will vary as we move from
receiving end towards the sending end. Vr
= sin hg X + Ir cos hg X
z r + jω L Zc
Zc and g are complex ⇒ Zc = = The above equations can be re-written for x = l, where
y g + jω c
V = Vs and I = Is
L
For a loss – less line, r = 0, g = 0, ⇒ Zc = Vs = Vr cos h g l + IrZc sin h g l
c
The Surge impedance (Zc) is purely resistive and is known sin h γ I
Is = Vr + I r cos h γ I
as surge impedance of the line. Zc
Comparing the above equations with standard equations,
Voltage Profiles
VR at No load
A = cos h g l
V(x)
B = Zc sin h g l
Vs sin h γ l
VR at SIL C= , D = cos h g l.
Zc
VR at Full Load We can see that A = D and
VR at Short circuit
x=1
x = 0 Receiving End AD – BC = cos h2 g I – sin h2 g I = 1
Sending End
Chapter 1  Power System Generation, Transmission and Distribution  |  3.881

Equivalent Representation of a Long Line Y ′Z′ Y ′Z′


1+ = cos h g l ⇒ 1 + × Z c sin h g l = cos h g l
For Line lengths more than 160 km, the parameters 2 2
are assumed to be distributed. The equivalent-T and (as Z = z l and g = yz )
Equivalent-p networks are represented. For long lines, nom-
inal-T and nominal-p networks may not be used as these Y′
⇒ Z sin h g l = cos h g l – 1
networks consider the lines with lumped parameters. 2 c
γl γl γl γl
Equivalent-p Representation =
cos h 2 + sin h 2 − cos h 2 + sin h 2
2 2 2 2
The sending end and receiving end voltages and currents
γl γl γl
are found form this equivalent network. ⇒ Y ′ Z c sin h cos h = 2 sin h 2
2 2 2
l1 Z′ l2
Y′ 1 γl y yzl tan h γ l / 2
⇒ = tan h = × ×
2 Zc 2 z 2 yz l / 2
V1 Y′ Y′ V2
2 2 Y tan h γ l / 2
= .
2 γ l /2
where ‘Y’ is the total shunt admittance of the nominal-p net-
V1 – Sending end voltage work. Hence, we can conclude that the total shunt admit-
V2 – Receiving end voltage tance of the nominal-p network should be multiplied with
I1 – Sending end current tan h γ L / 2
I2 – Receiving end current. γ L/2
Y ′Z′ ⎞ Hence, the equivalent circuit can be redrawn as
V1 = ⎛⎜1 + Vr + Z ′ I r (11)
⎝ 2 ⎟⎠ z(sin h gl)
Z ′=
l1 (g l) l2
Y ′Z′ ⎞ ⎛ Y ′ Z ′ ⎞ (12)
I1 = y ′ ⎛⎜1 + ⎟ Vr + ⎜1 + Ir
⎝ 4 ⎠ ⎝ 2 ⎟⎠
Y′ Y ′ = Y tan h g l /2
Comparing the above two equations with the standard ones, V1
2 2 2 g
i.e., 2
T
V1 = V2 cos h g l + Zc sin h g l
2
sin h γ l T Equivalent T-Representation of Long Line
I1 = V2 + cosh g l
Zc 2
l1 Z ′/2 Z ′/2 l2
Y ′Z′
⇒ 1 + = cos h g  l, Z′ = Zcsin h g l,
2
Y′
⎛ Y ′ Z ′ ⎞ = sin h γ l and 1 + Y ′ Z ′ = cos h g l V1 V2
Y ′ ⎜1 +
⎝ 4 ⎟⎠ Zc 2

Consider = Z   ′ = Zc sin h g l


V1 and V2 are the sending end and receiving end voltages,
z l respectively, whereas I1 and I2 are the sending end and
= × sin h g l × × yz
y l yz receiving end currents,respectively.
From the network of nominal-T,
sin h γ l sin h γ l We have,
= Z l =Z . ⎛ YZ ⎞ ⎛ YZ ⎞
γl γl VS = Vr ⎜1 + ⎟ + I r Z ⎜1 + 4 ⎟
⎝ 2 ⎠ ⎝ ⎠
From the above expression, we can observe that the equiv-
alent series impedance, the lumped impedance must be ⎛ YZ ⎞
IS = Y Vr + ⎜1 + Ir
sin h γ l ⎝ 2 ⎟⎠
­multiplied with .
γl As the above network is similar to nominal-T network, we
In order to get the shunt parameters of the equivalent-p can use the above equations to analyse the equivalent-T
network, substitute Z ′ from the above equation, i.e., network.
3.882 | Power Systems

⎛ Y ′Z′ ⎞ ⎛ Y ′Z′ ⎞ Therefore to get the series equivalent impedance, the


V1 = ⎜ 1 + ⎟ V2 + Z ′ ⎜1 + I2
4 ⎟⎠
lumped series impedance of the nominal-T network should
⎝ 2 ⎠ ⎝
γl
tan h
Y ′Z′ ⎞
I1 = ⎛⎜1 + I 2 + Y ′V2 be multiplied with 2 .
⎝ 2 ⎟⎠ γl
Comparing the above equations with the equations of long 2
The equivalent T-representation is as follows.
transmission lines, i.e.
V1 = Vs cos h γ l + I 2 Z c sin h γ l Z ′ Z tan h g l /2
=
l1 2 2 g l /2 l2
sin h γ l
I1 = V2 + I s cos h γ l
Zc Y sin h g l
Y ′=
V1 gl V2
⎛ YZ ⎞ ⎛ Y ′ Z ′ ⎞ = Z sin hg l
⎜⎝1 + ⎟ = cos hg l, Z ′ ⎜1 +
2⎠ 4 ⎟⎠
c

Y ′Z′ sin h γ l
1+ = cos hg l, Y ′ = .
2 Zc
To find the shunt branch parameters of the equivalent-T Calculation of Receiving End Voltages
circuit, and Currents
1 y yz × l We know that
Y ′ = sin h γ l = × sin h γ l ×
Zc z yz × l VS = AVr + B Ir(13)

sin h γ l IS = CVr + D Ir(14)


=Y (where g = yz).
γl
where Vs, Is are sending end voltage and current, respec-
Hence, to find the equivalent shunt admittance, the lumped tively, while Vr, Ir are receiving end voltage and current,
sinh γ l respectively.
shunt admittance should be multiplied with .
γl Multiply (13) with C and (14) with A
To find the series impedance, consider the equation. ⇒ CVS = VA Vr + CB Ir(15)
⎛ Y ′Z′ ⎞
⎜1 + 2 ⎟ = cosh g l AIS = AC Vr + AD Ir(16)
⎝ ⎠
Subtract (15) from (16),
sin h γ l
Substitute Y ′ = in the above equation,
Zc AIS – CVS = AC Vr + AD Ir – CAVr – EBIr

Z 1 sin h γ l ⇒ AIs – CVs = Ir(AD – BC)


⇒ 1+ × = cos h g l
2 Zc We know that AD – BC = 1 and A = D, receiving end current
γl γl is expressed as
2 sin h cos h
Z′ 2 2 = cos h g l – 1 = 2 sin h2 γ l
⇒ × ⇒ Ir = –CVS + DIS(17)
2 ZC 2
Z′ γl γ γl Multiply (13) with ‘D’ and (14) with ‘B’
⇒ = Z C tan h = tan h
2 2 y 2 DVs = AD Vr + DB Ir(18)
yz .l
Multiply and divide by BIs = BC Vr + BD Ir (19)
2
Subtract (18) from (19),
Z′ z yz .l tan h ( γ l / 2 )
⇒ = × ×
2 y 2 yz .l DVs – BIs = (AD – BC) Vr
2
We know that AD – BC = 1, receiving end voltage is
γl expressed as
tan h
Z 2 (where z = x.l and g = zy)
=
2 γl ⇒ Vr = DVs – BIs(20)
2
Chapter 1  Power System Generation, Transmission and Distribution  |  3.883

Constants of Two Networks in Series l11 l21


l1 A1 B1 l2
Two networks are said to be connected in series when the l22
output of one network is connected to the input of the other C1 D1
V1 V2
network. Let the constants of the, network be A1, B1, C1, D1
and the constants of the networks be A2, B2, C2, D2, provided
there are two networks connected is series as shown in the
below figure.
l12
I1 I2 A2 B2
I
A1 B1 A2 B2 C2 D2
V1 V V1
C1 D1 C2 D2
From the above network, we can write that
From the figure, V1 = A1V2 + B1I21(25)
V = D1V1 – B1I1
V1 = A2V2 + B2I22(26)
I = –C1V1 + A1I1
And, V = A2V2 + B2I2 And the combination of the two networks as a single net-
work can be written is the form of equations as
I = C2 V2 + D2I2
From the above equations, we can see that V1 = AV2 + BI2
D1V1 – B1I1 = A2V2 + B2I2(21) where I2 = I21 + I22
Multiply Equation (25) with B2 and (26) with B1 and adding
–C1V1 + A1I1 = C2V2 + D2I2(22) these results in
Multiply Equation (21) with A1 and (22) with B1 and adding V1B2 = A1B2 V2 + B2B1I21
the equations,
+ V1B1 = A2B1V2 + B1B2I22
A1D1V1 – A1B1I1 = A1A2V2 + A1B2I2 –––––––––––––––––––––––––––––––––––––
+ –B1C1V1 + A1B1I1 = B1C2V2 + B1D2I2 V1(B1 + B2) = (A1B2 + A2B1)V2 + B1B2 (I21 + I22)
––––––––––––––––––––––––––––––––––––––––––
V1(A1D1 – B1C1) = (A1 A2 + B1 C2)V2 + (A1B2 + B1D2)I2(23) A1 B2 + A2 B1 BB
⇒ V1 = V2 + 1 2 I 2 (27)
B1 + B2 B1 + B2
Multiply (21) with C1 and (22) by D1 and the equations,
From Equation (27), we can infer that
C1D1 V1 – B1C1I1 = A2C1V2 + B2C1I2 A1 B2 + A2 B1 B1 B2
+ –C1D1V1 + A1D1I1 = C2D1V2 + D1D2I2 A= and B =
B1 + B2 B1 + B2
––––––––––––––––––––––––––––––––––––––––––––
I1(A1D1 – B1C1) = V2(A2 C1 + C2D1) + I2(B2C1 + D1D2)(24) As the transmission line in a symmetric network,
A1 B2 + A2 B1 D1 B2 + D2 B1
We know that A1D1 – B1C1 = 1 A=D= =
B1 + B2 B1 + B2
Equations (23) and (24) represent the combination of two
network and representing as a single network with a single And AD − BC = 1 and substituting A, B, D in this equation,
constants. we get
\ A = A1A2 + B1C2 ⎛ A1 B2 + A2 B1 ⎞ ⎛ A1 B2 + A2 B1 ⎞ B1 B2
⇒ ⎜⎝ B + B ⎟⎠ ⎜⎝ B + B ⎟⎠ − B + B × C
B = A1B2 + B1D2 1 2 1 2 1 2

C = A2C1 + C2 D1
D = B2C1 + D1D2 ⇒ C = C1 + C2 +
( A1 − A2 ) ( D2 − D1 )
Therefore the above relations represented in matrix form as B1 + B2

⎡ A B ⎤ ⎡ A1 B1 ⎤ ⎡ A2 B2 ⎤ Example 34:  A 220 kV, 20 km long, 3-f transmission line
⎢C D ⎥ = ⎢C D1 ⎥⎦ ⎢⎣C2 D2 ⎥⎦ has the following A, B, C, D constants.
⎣ ⎦ ⎣ 1
A = D = 0.96∠3o
B = 55 ∠65o Ω/phase
Constants for Two Networks in Parallel C = 0.5 × 10–4 ∠90o S/phase
The two networks connected in parallel are as shown below Its charging current per phase is
and the constants for the combination of two networks are (A) 6.3 A (B) 11 A
calculated as follows. (C) 127 A (D) 220 A
3.884 | Power Systems

Solution:  (A) 1.1


C/km =
Admittance per phase (Y) = C 2π × 50 × l
=
0.5 × 10−4 ∠90o
Charging current per phase = Vpn × Y 1
Velocity of propagation =
220 LC
= × 103 × 0.5 × 10 −4
3 l
11 30 × 105 =
= A = 6.35 A 0.04 1.1
3 ×
2π × 50 2π × 50
Example 35: Two networks are connected in cascade as
shown in figure with the usual notation the equivalents A, B, l = 2124.5 km
C and D constants are obtained. Given that C = 0.025 ∠40o,
the value of Z2 is
Example 37:  A single-phase 50 Hz generator supplies an
Z1 = 10 ∠30° inductive load of 5 MW at a power factor of 0.7 lagging by
means of an overhead transmission line 20 km long. The
line resistance and inductance are 0.019 Ω and 0.62 mH
Z2
per km. The voltage at receiving end is required to be kept
at 10 kV.
Calculate
(A) 40∠40o (B) 20∠40o 1. The sending end voltage and voltage regulation of the
(C) 10∠20o (D) 20∠20o line.
2. The value of capacitors to be placed in parallel with the
Solution:  (A)
load such that the regulation is reduced to 60% of that
For the given two part network obtained in the previous problem.
1 3. Calculate the transmission efficiency in case (i) and (ii).
The open-circuit admittance =
Z2
1 Solution:  Given transmission line is 20 km, so it is consid-
∴C = ered as a short line.
Z2
1 \ I = IR = IS given by
Z2 = = 40 ∠ − 40°Ω
C
5 × 103 P
IR = = = 714 A
Example 36:  The total reactance and total susceptance of a 10 × 0.7 V cos ϕ
lossless overhead EHV line operating at 50 Hz are given by
0.04 p.u. and 1.1 p.u., respectively. If the velocity of wave Source voltage
propagation is 30 × 105 km/s, then the approximate length
|Vs| = |VR| + |I|(RcosfR + XsinfR)
of line is
(A) 200 km (B) 2000 km
 =
10,000 + 714(0.38 × 0.7 + 3.89 × 0.714)
(C) 20 km (D) 2 km
Solution:  (B) Sending end voltage = 12.173 kV
1
Velocity of wave propagation (J ) = 12.173 − 10
( L/km) / (C/km) Voltage regulation = × 100
10
Total reactance of line (XL) = 0.04 p.u.
= 21.73%
XL 0.04
L/km = =
2π fl 2π × 50 × l (ii) Desired voltage regulation = 60% of 21.73

Total susceptance of line = 1.1 p.u. = 0.6 × 21.73 = 13.03%
1
= 2π fC = 1.1 p.u. Vs − 10
Xc
Regulation = 0.13
10
1.1
C= Vs = 11.3 kV
2π × 50
Chapter 1  Power System Generation, Transmission and Distribution  |  3.885

lR
R + jx
l XL = 2πfL = 2π × 50 × 1.21 × 10–3 × 200 = 76.02 Ω

Z = R + jXL = 20 + j76.02 = 78.6 ∠75.2o


lc
Vs Load Y = jwcl = j.314 × 0.0096 × 10–6 × 200

Y = 6.02 × 10–4 ∠90o


25 × 1000
Source voltage in the above circuit after placing capacitor to IR = ∠31.78 = 109.3∠ − 31.78
improve voltage regulation 3 × 132

Vs = Vr + IR(Rcosfr + X sinfr) 132


VR ( per phase ) = ∠0° = 76.2∠0° kV
Vs – Vr = 0.13 kV = IR(R cosfr + X sinfr) 2

Since capacitance does not draw any real power


lS Z lL lR
5000
IR =
10 × cos ϕR

From the above two equations, cosfr = 0.8685, IR = 575.7 A
Now capacitor current Ic = IR – I VS Y Y VR
2 2
=
575(0.8685 – j0.495) – 714(0.7 – j0.714)

= 0.195 + j225.17
Figure 15  Medium line p–representation

Capacitor current IC = j225.17 A


Source voltage
1 V 10 × 1000
Xc = = R = ⎛ 1 ⎞
ωc I C 225.17 Vs = ⎜1 + YZ ⎟VR + ZI R
⎝ 2 ⎠
225.17 225.17
C= = ⇒ C = 71.71 μF ⎡ 1 ⎤
10 4 × ω 10 4 × 314 = ⎢1 + × 6.02 × 10 −4 ∠90 × 78.6 ∠75.2⎥ × 76.2
(iii) Efficiency of transmission ⎣ 2 ⎦
Output + 78.6 ∠75.2 × 109.3 ∠–31.78 × 10–3
η=
Output + Losses = 76.2 + 1.8 ∠166.2 + 8.59 ∠43.42

5000 VS = 80.69 + 6.3j = 80.93 ∠4.48o
Case (i): η = = 0.962%
5000 + 714 2 × 0.38 × 10 −3 Vs ( line ) = 80.93 × 3 = 140.17 kV
5000
Case (ii): η = = 97.5% 1 1
500 + 5755 × 0.38 × 10 −3 1 + Yz = 1 + × 6.02 × 10 −4 ∠90 × 78.6 ∠75.2
2 2
Example 38:  A 200 km, 3-f, 50 Hz transmission line de- = 1 + 0.023 ∠165.2 = 0.977 + j0.006
livering 25 MVA at 0.85 lagging power factor to a balanced
140.17
load at 132 kV. The line conductors are spaced equilater- VR 0 ( line no load ) = = 143.47 kV
ally 2.5 m apart. The conductor resistance is 0.1 Ω/km and 0.977
its effective diameter is 1.5 cm. Calculate the sending end
143.47 − 140.17
voltage and voltage regulation. Voltage regulation = = 2.4%
140.17
Solution:  Transmission line parameters
D 250 Example 39:  A 50 Hz, 250 km long transmission line has
L = 0.461 log 1 = 0.461.log. = 1.21 mH/km
 r 0 .778 × 0.75 a total series impedance of 30 + j 100 Ω and a total shunt
admittance of 10–3 Ω. The receiving end load is 40 MW at
0.0242 0.0242
C= = = 0.0096 μF/km 220 kV with 0.8 lagging power factor. Calculate the sending
D 250 end voltage, current, power and power factor using.
log log
r 0.75
(i) Short line approximation
R = 0.1 × 200 = 20 Ω (ii) Nominal π-method
3.886 | Power Systems

Solution:  Impedance of the line Z = 30 + j100 Ω Underground Cables


= 104.4 ∠73.3
Underground transmission has more advantages over over-
Y = 10–3 ∠90o 
head transmission like low maintenance, cost, good look-
The receiving end load is 40 MW at 220 kV, 0.8 p.f. lagging
ing, free from damage due to lightning and thunder storms
40 and has no interference with communication circuit. The
∴ IR = ∠ − 36.9°
3 × 220 × 0.8 underground transmission is done by underground cables.
The cable used in underground transmission generally
220
= 0.1312∠ − 36.9°kA VR = ∠0° = 127∠0° kV has three parts.
 3
1. The core (or) conductor.
(i) Short line approximation 2. The insulating material.
Vs = VR + IRZ = 127 + (0.164 ∠−36.9 × 104.4 ∠73.3) 3. A metallic sheath of lead is provided around the insu-
= 127 + 17.12 ∠36.4 lation to provide external protection.
= 141.14 ∠4.127 o
The characteristics of insulating material used in cables are
= 140.7 + 10.15j
(i) High dielectric strength.
Vs line = 244.46° kV (ii) Good mechanical properties and the insulating mate-
Is = IR = 0.1312 ∠–36.9 rial should not affected by chemicals.
Sending end power factor (
iii) High insulation resistance.
=
cos(4.127 + 36.9) = 0.754 lagging Depend upon the operating voltage and temperature, dif-
ferent types of insulating materials used in cables those are
Sending end power = 3 × 244.4 × 0.1312 × 0.754 listed as follows.
= 41.87 MW
(i) Polyvinyl chloride (PVC)
( ii) Nominal-π method (ii) Impregnated paper
1 1 (iii) Vulcanized rubber
A = D = 1 + YZ = 1 + × 10 −3 ∠90 × 104.4∠73.3°
2 2 (iv) Polythene
= 1 + 0.0522 ∠163.3 = 0.95 ∠0.90o
B = Z = 104.4 ∠73.3o Classification of Cables
⎛ 1 ⎞ ⎛ 1 ⎞ The underground cables are classified depending upon the
C = Y ⎜1 + YZ ⎟ = Y ⎜1 + × 10 −3 ∠90 × 104 : 4∠73.3 ⎟
⎝ 4 ⎠ ⎝ 4 ⎠ operating voltages are

1. Low-tension cable (LV-cable): These cables are gen-
= 9.75 × 10–4 ∠90.4o
erally used for an operating voltage below 1 kV.
Vs = AVR + BIR = 0.95 ∠0.90 × 127 + 2. High-tension cable (HV-cable): These cables are used
104.4 ∠73.3 × 0.13 ∠–36.9o for an operating voltage below 11 kV.
3. Super tension cable (ST-cable): These cables are used
=
120.6 ∠0.9 + 13.57 ∠36.4 for an operating voltage up to 33 kV.
Vs = 131.88 ∠4.32 4. Extra high-tension cable (EHT-cable): These cables
are used for an operating voltage of 66 kV.
|Vs|line = 228.4 kV 5. Extra super tension cable: These cables are used for
Is = CVR + DIR an operating voltage of above 132 kV.

=
9.75 × 10–4 ∠90.4 × 127 + 0.95 ∠90o
× 0.13 ∠–36.9o Dielectric Stress in Cable
= 0.123 ∠90.4 + 0.1235 ∠–36 In a cable the current carries in a core (or) conductor of the
cable. So, the stress is more on the surface of the core and
Is = 0.111 ∠26.96o it decreases towards the surface of the cable (or) sheath of
Sending power factor = cos(26.96 – 4.32) = cos 22.64 = the cable.
0.922 leading Maximum gradient at the surface of the conductor is
V
Sending end power g max =
⎛ R⎞
= 3 × 228.4 × 0.111 × 0.922 = 40.5 MW r ln ⎜ ⎟
⎝ r⎠
Chapter 1  Power System Generation, Transmission and Distribution  |  3.887

Conductor In cable if the rate of heat generation is more than the rate of
heat dissipation, then the temperature of the cable Increases.
The sources of heat generation in cables are
1. Dielectric loss
r 2. Copper loss in the core
R
3. Sheath losses

Sheath
Corona
Dielectric The ionization of air surrounding the power conductor is
Minimum gradient at the sheath of the cable corona. There are free electrons normally present in the free
V space due to radioactivity and cosmic rays. The gradient
g min = around the surface of the conductor increases with increases
⎛R⎞
R ln ⎜ ⎟ in the potential between the conductors. The free electrons
⎝r⎠ present in the air will move with certain velocity depending
Here, V is the voltage applied to the cable on the field strength. These moving electrons collide with
r – Radius of the conductor (or) core the molecules of the air and some of the fast moving elec-
R – Radius of the cable trons may also dislodge electrons from the air molecules,
thereby increasing the number of electrons. Hence the pro-
Grading of Cable cess of ionization is thus cumulative and may lead to elec-
tron avalanche. Hence the air surrounding the conductor
Grading of cable is done according to the stress on the
may be ionized.
dielectric material of the cable. There are two methods of
When a voltage higher than the critical voltage is applied
grading.
between two parallel polished wires, the glow is quite obvi-
1. Capacitance grading ous. After it is operated for some time, reddish heads or tufts
2. Inter sheath grading form along the wire, bluish white glow is appeared around
the surface of the wire.
Capacitance Grading It is noticed that the reddish tufts or beads are formed
when the conductor is negative, and a smoother bluish
In this method, different dielectric materials having differ- white glow is formed when the conductor is positive. The
ent permittivities are used and these materials are used in corona due to AC is same as the corona due to DC when
cables by two ways. viewed through stroboscope. Due to corona, a hissing noise
1. The factor of safety for all the materials is same, i.e., is observed and ozone gas is formed which is detected by its
the material which has highest product of dielectric characteristic odour.
strength and permittivity should be placed near to the
core of the cable. e –– – –
e– e e –
R
2. The same working stress for different materials but the e– e
e– e – Other molecules
e– e–
material which has highest permittivity placed near to e– e e
– –
e–
the conductor of the cable. e–
e–
e– Electron Avalanche
E e
max e –– y
Intersheath Grading e–
e–
In this method, different metallic sheaths are used in e
Ir e –– Ionosed Arc (L–L fault)
between the core and sheath of the cable for the same die- e–
e–
lectric material by applying different voltages to different e–
E e B
metallic sheath from operating voltage to earth potential. max
EFI is maximum at
So, the stress on the same material also changed corre- surface of the conductor
sponding to the voltage.

Laying and Heating of Cables Overhead transmission lines are used for all economic
Generally there are three methods of laying of cables reasons than underground cables. If line-to-line operating
voltage ≥ 275 kV, then there will be abnormal electric field
1. Draw-in-system intensity (EFI) at the surface of the conductor.
2. Direct laying EFI (increase or decrease a potential difference a mobil-
3. Solid system ity of electrons).
3.888 | Power Systems

Critical Disruptive Voltage πε 0 V 1 d V d


⇒ E= × × = ×
Consider a single-phase transmission line. Let ‘r’ be the d 2πε 0 x ( d − x ) d x (d − x )
ln 2 ln
radius of each conductor and ‘d’ be the distance of separa- r r
tion such that ‘d’ is far greater than ‘r’. Let ‘q’ be the charge
V
per unit length of the conductor on one of the conductors Let V ′ = which is line to neutral voltage. In case of three-
and hence ‘–q’ is on the other. If the operating voltage is 2
V
‘V’, the potential of conductor ‘A’ with respect to neutral phase system, V ′ = , where ‘V’ is line-to-line voltage
3
V −V
plane ‘N’ will be and that of ‘B’ will be ‘ ’. Consider V ′d
2 2 ⇒ E=
a point ‘p’ at a distance ‘x’ where we want to find the electric d
x ( d − x ) ln
field intensity. r
Neutral plane From the above expression, it is clear the EFI is inversely
N proportional to ‘x’, i.e. as ‘x’ decreases, EFI increases and it
is maximum when x = r, i.e., at the surface of the conductor
A and this value i given by
p B
q V ′d V′ d
–q Emax = ≈ ⇒ V ′ = Emax r ln
x d d r
( d − r ) r ln r ln
d
r r
Critical disruptive voltage is hence defined as the voltage
at which the complete disruption of dielectric occurs. This
The field due to ‘A’ will be repulsive and due to ‘B’ would voltage gives the gradient at the surface of the conductor
be attractive by bringing a unit positive charge from infin- equal to the breakdown strength of the air. This dielectric
ity to that point ‘p’. Hence the EFI’s due to A and B at ‘p’ strength is normally denoted by g0 and is equal to 30 kV/
would be additive cm peak at normal temperature and pressure, i.e. 25°C and
q q q ⎡1 1 ⎤ 76 cm of Hg. At any other temperature and pressure,
E= + = +
2πε 0 x 2πε 0 ( d − x ) 2πε 0 ⎣ x d − x ⎥⎦

g0′ = g0 × d
The potential difference between the conductors,
where ‘d ’ is the air density correction and is given by
r d −r
q ⎡1 1 ⎤ 3.92 b
V=– ∫
d −r
E dx = ∫ r
+
2πε 0 ⎣ x d − x ⎥⎦
⎢ dx d=
273 + t
where ‘b’ is the barometric pressure in cm of Hg and ‘t’ the
q temperature in °c.
= ⎡ln x − ln ( d − x )⎤⎦ r d − r
2πε 0 ⎣ Hence, the critical disruptive voltage is given by
d
q V ′ = rg0d ln
kV.
 = ⎡⎣ln ( d − r ) − ln ( d − ( d − r ) ) − ln r + ln ( d − r ) ⎤⎦ r
2πε 0
where the gradient ‘g’ is nothing but electric field intensity
(EFI).
q ⎡d − r⎤ q ⎡d − r⎤
= × 2 ln ⎢ ⎥ = ln ⎢ ⎥ The above expression is derived based on the considera-
2πε 0 ⎣ r ⎦ πε 0 ⎣ r ⎦ tion that the conductor is solid and the surface is smooth.
as d > > r, d – r @ d, ACSR conductors are used for higher voltages. The poten-
tial gradient for ACSR conductors will be greater than the
q d potential gradient of an equivalent smooth conductor, and
⇒  V = ln
πε 0 r hence the breakdown voltage of a stranded conductor will

Now the EFI at the point, which is ‘x’ distance from the be comparatively less than a smooth and solid conductor.
centre of the conductor ‘A’ is given by, The irregularities on the surface of such a conductor are
increased further due to the deposition of dust and dirt
q ⎡1 1 ⎤ q d on its surface and hence the breakdown voltage is further
E= + = .
2πε 0 ⎢⎣ x d − x ⎥⎦ 2πε 0 x ( d − x ) reduced.
Hence, the final expression for the critical disruptive
From (1), substitute ‘q’ in above equation.
voltage after considering atmospheric conditions and the
πε 0 V surface of the conductor is
q =
d d
ln V ′ = rg0dm0 ln kV.
r r
Chapter 1  Power System Generation, Transmission and Distribution  |  3.889

When the voltage applied corresponds to the critical disrup-


= 2 × 21.2 × 0.951 × 0.9 × ln ⎛⎜
200 ⎞
⎝ 2 ⎟⎠
tive voltage, corona creates but may not be visible as the
charged ions in the air must receive some finite energy to
cause further ionization by collision. For a radial field, it =
167.28 kV/ph
must reach a gradient gv at the surface of the conductor to \ Line voltage (RMS) = 3 × 167.28
cause a gradient g0, a finite distance away from the surface
=289.72 kV
of the conductor. ‘gv’ is not constant as ‘g0’ is, and is a func-
tion of the size of the conductor. Example 41:  A 132 kV line with 2 cm diameter conduc-
⎛ 0.3 ⎞ tors is built so that corona takes place if the line voltage
gv = g0d ⎜1 − ⎟ kV/cm exceeds 220 kV (RMS). If the value of potential gradient at
⎝ rδ ⎠ which ionization occurs can be taken as 30 kV per cm, find
If ‘Vv’ is the critical visual disruptive voltage, then the spacing between the conductors.
d
Vv = gvr ln Solution:  If the line is 3-f, Radius of the conductor, r =
r 2 30
Vv ⎛ 0.3 ⎞ = 1 cm. Dielectric strength of the air, g0 = = 21.2
⇒ gv = = g 0δ ⎜ 1 + ⎟ 2 2
d ⎝ rδ ⎠ 220 kV
r ln kV (RMS). Disruptive voltage/phase = = 127.020
r 3
kV. If the conductors are assumed to be smooth conductors,
⎡ 0.3 ⎤ d
⇒ Vv = rg0 d ⎢1 + ⎥ ln r kV then irregularity factor m0 = 1. At standard temperature and
⎣ rδ ⎦ pressure, air density factor d = 1
In case the irregularity factor is taken into account, ⎛ d⎞
\ V ′ = rg0dm0 ln ⎜ ⎟ kV
⎡ 0.3 ⎤ d ⎝ r⎠
Vv = g0mvdr ⎢1 + ⎥ ln r
⎣ rδ ⎦ where ‘d’ is spacing between the conductors.
⎛ d⎞
127.020 = 1 × 21.2 × 1 × 1 × ln ⎜ ⎟
⎡ 0.3 ⎤ d ⎝ 1⎠
21.1 mv dr ⎢1 +
= ⎥ ln kV RMS
⎣ rδ ⎦ r ⎛ d⎞
⇒ ln ⎜ ⎟ = 5.9915 ⇒ d = 400.014 cm
⎝ 1⎠
where ‘r’ is the radius in cm. The irregularity factor mv has
the following values:
Corona Loss
mv = 1.0 for polished wires
The electric field produces ions in the space which fur-
=
0.98 to 0.93 for rough conductor exposed to atmos- ther leads to the production of space charge which move
pheric conditions around the conductor. If there is a continuous transmission
of energy, there is a continuous motion for these charges.
= 0.72 for local corona on stranded conductors
Hence, it can indirectly be said that the energy required for
Since the surface of the conductor is irregular, the corona the charges to remain in motion is derived from the sup-
does not start immediately on the whole surface but it takes ply system. In order to maintain the flow of energy over
place at difference points of the conductor which are pointed the conductor in the field where in this additional energy
and this is known as local corona. would have been otherwise absent, it is necessary to supply
Example 40:  A three-phase line has conductors 4 cm in this additional loss from the supply system. This additional
diameter spaced equally 2 m apart. If the dielectric strength power is referred as corona loss.
of the air is 30 kV (max) per cm, find the disruptive critical An empirical relation was derived by peek by conducting
voltage for the line. Take air density factor, d = 0.951 and a number of experiments on corona loss.
irregularity factor = 0.9.
( f + 25)r
(VP − VC ) ×10 −5 kW/km/phase
2
4 P = 241×
Solution:  Conductor radius = = 2 cm δ d
2
Conductor spacing = 2 m = 200 cm where ‘f’ is the supply of frequency,
Dielectric strength of the air, ‘d’ – Air density correction factor,
g0 = 30 kV/cm (max) Vp – The operating voltage in kV
V0 – Critical disruptive voltage
=
21.2 kV (RMS) per cm
3.92 b
and d= ,
Disruptive critical voltage 273 + t
⎛ d⎞ where b = Atmospheric pressure
V ′ = rg0d m0 ln ⎜ ⎟ kV.ph
⎝ r⎠ t = Temperature.
3.890 | Power Systems

The above equation is for a fair weather condition. For a foul Corona loss α (f + 25)
weather condition, the power loss can be obtained by taking P2 f + 25
V0 as 0.8 times the fair weather value. The above empiri- = 2
P1 = f1 + 25
cal formula has certain limitations and is applicable only if
the frequency is in between 25 and 120 Hz, the conductor 60 + 25
P2 = × P1 = 1.36 kW/km/ph
Vp 50 + 25
radius is greater than 0.25 cm and the ratio of > 1.8.
V0
Example 42:  A 3-f 220 kV, 50 Hz transmission line con- Factors Affecting Corona Loss
sists of 3 cm diameter conductor spaced 4 m apart in an The factors that affect corona loss on overhead transmission
equilateral triangle form. If the temperature is 40°C, atmos- lines:
pheric pressure is 76 cm, calculate the corona loss per km
of the line. Take m0 = 0.9. 1. Electrical factors
2. Atmospheric factors and
Solution:  We know that 3. Factors with respect to conductors
( f + 25) r
P = 241 × 10–5 (Vp – V0)2 kW/km/phase Electrical Factors
δ d
According to the above power loss expressions corona loss
3.92 × b 3.92 × 76 is a function of frequency. Hence if the frequency increases,
d = = = 0.952
273 + t 273 + 40 the corona loss also increases proportionately. Hence we
g0 = 21.2 kV/cm (RMS) can infer that DC corona loss is comparatively less than AC
corona loss. Hence if in case of AC supply, there may be
Critical disruptive voltage per phase harmonic which may further increase the corona loss.
d
V0 = m0g0 d r loge kV
r Field Around the Conductor
200
V0 = 0.9 × 21.2 × 0.952 × 1.5 × loge The field around the conductor is a function of the voltage.
1.5 Along with .voltage, field also depends on the configuration
=
27.24 × 4.80 = 133.281 kV of the conductors. The height of the conductors from the
ground also effects the corona loss. The smaller the height,
Supply voltage per phase,
the greater will be the corona loss.
220
Vp = = 127 kV
3 Atmospheric Factors
(50 + 25) 1.5 Pressure and Temperature Effect
\ P = 241 × 10–5 × × × (127 – 133.281)2 From the expression for power loss, we can observe that it is
0.952 200
kW/km/phase a function of air density correction factor ‘d ’ and even critical
disruptive voltage is a function of ‘d ’. The lower the value of
=
0.01644 × 39.450 = 0.648 kW/km/phase ‘d ’, the higher is the loss. For lower values of ‘d ’, the pressure
should be low and the temperature should be high. Hence the
\ Total corona loss per km for the three phases corona loss is low in plain areas and high in hilly areas.
= 3 × 0.648 = 1.944 kW. Dust, Rain, Snow and Storm Effect
The particles of dust deposit on the conductor, thereby
Example 43: Which of the following will minimize the reducing the critical disruptive voltage for local corona and
corona loss? also the rains, snow and hail storm reduce the critical dis-
(A) Reduction in conductor size ruptive voltage and the corona loss increases.
(B) Reduction in smoothness of the conductor
(C) Sharp points are provided in the line hardware Factors with Respect to Conductor
(D) Current density is conductor is reduced.
Diameter of the Conductor
Solution:  (D) From the expression, we can see that the corona loss is
Example 44:  The corona loss on a particular system at 50 directly proportional to the radius of the conductor and also
Hz is 1.2 kW/km/phase. The corona loss at 60 Hz would be we know that loss a (Vp – V0)2, where V0 is the critical dis-
(A) 1.2 kW/km/ph (B) 1.36 kW/km/ph ruptive voltage which is inversely proportional to the radius
(C) 1.14 kW/km/ph (D) 1.42 kW/km/ph of the conductors. Hence, the larger the size of the conduc-
tor, the lower is the corona loss.
Solution:  (B)
Chapter 1  Power System Generation, Transmission and Distribution  |  3.891

Number of Conductors/phase the dielectric strength of air to be 21.1 kV/cm, the corona
If the operating voltage ≥ 380 kV, it is found that using one losses will be (m = 0.75)
conductor per phase produces large coroner loss and also (A) 5 kW/km (B) 10 kW/km
large radio interference in the communication lines running (C) 15 kW/km (D) 20 kW/km
parallel to power lines. Hence, using ‘2’ or more conductors
per phase reduces the corona loss, i.e. using bundled con- Solution:  (C)
ductors. By using bundled conductors, the self-GMD of the 220
Phase voltage = kV = 127 kV
conductor reduces increases the critical disruptive voltage 3
and thereby the corona loss reduces.
Critical disruptive voltage
Heating of the Conductor by Load Current
⎛ d⎞
The current passing through the conductor produces I 2R (E0) = mg0δ ⎜ ln ⎟ kV/phase
losses which would help to reduce the corona loss. If there ⎝ r⎠
is no such heat produced, then the conductor’s temperature ⎛ 300 ⎞
= 0.75 × 1.05 × 21.1 In ⎜
would be slightly less than the surrounding temperature. In ⎝ 0.65 ⎟⎠
the absence of heating, due to high humidity at times, addi- = 101.93 ≈ 102 kV
tional corona is introduced.
Critical disruptive voltage E0(L – L)
Methods of Reducing Corona Loss = 3 × 102 = 176.6 kV
The losses can be reduced by
Corona loss in the system is given by
1. Large diameter conductors
2. Hollow conductors ⎛ f + 25 ⎞ r
( E − E0 ) kW/phase/km
2
3. Bundled conductors P = 241 × 10–5 ⎜ ⎟
⎝ 8 ⎠ D
4. By increasing the conductor’s spacing
75
= 241 × 10–5 × × 29.09
Advantages and Disadvantages of Corona δ
Advantages = 5.007 kW/phase/km
1. Due to corona formation, the air surrounding the con- = 15.02 kW/km
ductor also becomes conducting and therefore virtu-
ally the diameter of the conductor increases. This
increased diameter reduces electrostatic stresses be-
tween the conductors. Skin Effect
2. Corona reduces the effects of transients produced by When a current-carrying conductor is carrying direct cur-
surges by dissipating them as corona loss. rent (DC), the current is uniformly distributed over the
whole cross section of the conductor. But, if an alternating
Disadvantages current is flowing through the conductor, the current does
not distribute uniformly, instead, it is concentrated more on
1. Corona reduces the energy to be transmitted thereby
the surface of the conductor compared to the current flow-
reducing the transmission efficiency of the transmis-
ing through the centre of the conductor. This is known as
sion line.
skin effect.
2. The ozone produced during the corona may cause
The tendency of the alternating current to concentrate
corrosion of the conductor.
near the surface of a conductor is known as skin effect.
3. The current drawn by the conductor during the time
Due to skin effect, the effective area of cross section of
of corona is non-sinusoidal and produces non-sinu-
the conductor through which the current flows is reduced.
soidal voltage drop which may cause inductive inter-
Hence, the resistance of the conductor increases while car-
ference with neighbouring communication lines.
rying an alternating current. The conductor consisting of
4. Due to corona, the effective capacitance of the con-
a large number of strands each carrying part of the total
ductors is increased as the effective diameter of the
current. The inner strands carrying currents give rise to
conductor is increases. The charging current flowing
flux which link only the inner strands whereas the strands
also increases.
outside produce flux which links both inner and outside
Example 45:  A 3-f line is operating at 220 kV and 50 Hz. strands. The flux linkages per ampere are more for the inner
The line has a conductor of 1.3 cm diameter arranged in a strands than for the outer strands. Thus the inductance as
3 m delta connection. Assume air density factor of 1.05 and well as impedance of the inner strands is more than that of
3.892 | Power Systems

the outer strands, resulting in more current to flow in outer Ferranti effect can be explained by approximating the dis-
strands. Skin effect depends on the following factors. tributed parameters of the long transmission line by lumped
1. Nature of the material impedance as shown below.
2. Radius of the conductor
R L Ic
3. Frequency of supply
4. Shape of wire
Skin effect is more if the frequency of supply is more and is
proportional to the size of the conductor. V1 C V2

Proximity Effect
The alternating magnetic flux produced in the conductor
due to current flowing through the neighbouring conductor
gives rise to circulating currents which apparently increases
V2 Ic X
the resistance of the conductor. This phenomenon is called IC
Ic R
proximity effect. Consider a two-wire system in which
the lines of flux link with inner and outer elements of the
conductor but most of them link with the elements farther Vr
apart compared to the inner elements. Hence the inductance
of the farthest elements are more compared to the nearer As the capacitive reactance of the line is more than the
elements, thereby impedance of the farther elements are inductive reactance, under no load condition the line current
more compared to nearer elements. This gives raise to non- is of leading power factor. The charging current produces
uniform distribution of current. The effective resistance is drop in the reactance of the line which is in phase opposi-
thereby increased due to non-uniform distribution of cur- tion to the receiving end voltage and therefore the sending
rent. The proximity effect is more in case of underground end voltage becomes smaller than the receiving end voltage.
cables, where the distance between overhead line transmis- This is shown in the above phasor diagram.
sion lines is more Proximity effect depends on conductor’s
size, frequency, distance between conductors and perme- Example 46:  The surge impedance of a 200 km long over-
ability of conductor material. head line is 200 Ω. For a 100 km length of the same line, the
surge impedance will be
Ferranti Effect (A) 200 Ω (B) 100 Ω
(C) 400 Ω (D) 150 Ω
A long-transmission line operated under no load or lightly
loaded condition, the receiving end voltage is more than the Solution:  (A)
sending end voltage. This phenomenon is known as Ferranti
Surge impedance is independent of length of the line
effect. This is explained as follows.
We know the equation of voltage Example 47:  The reflection co-efficient for the transmis-
V + I r Z c α X j β X Vr + I r Z c −α X − j β X sion line shown in figure at ‘P’ is
V= r e e + e e
2 2 Z0 = 300 Ω P
under no-load condition, Tr Line
Load
The above equation reduces to
When x = L, Ir = 0 400 Ω

Vr aL jbL Vr –aL –jbL


Vs = e e + e e
2 2
V V (A) 0.25 (B) 0.142
At L = 0, Vr = r + r (C) 0.33 (D) 7
2 2
As ‘L’ increases, the incident component of sending Solution:  (B)
end voltage increases exponentially and turns the vector Z L − ZS
­anticlockwise through an angle bL whereas the reflected Reflection co-efficient =
Z L − ZS
sending end voltage decreases by the same amount and is
rotated clockwise through the same angle bL. The sum of 400 − 300
these two components of sending end voltage gives a volt- = = 0.142
700
age which is smaller than the receiving end voltage Vr.
Chapter 1  Power System Generation, Transmission and Distribution  |  3.893

Example 48:  An overhead line having a surge impedance Shunt Reactors


of 400 Ω is connected in series with an underground cable In this method, inductors are connected across the load.
having a surge impedance of 80 Ω. If surge of 40 kV travels Shunt reactors generally used when the bus voltage more
from the line towards the cable junction, the value of the than the specified voltage, i.e., under light load condition
transmitted voltage wave at the junction is ((or) Ferranti effect).
(A) 160 kV (B) 40 kV
(C) 80 kV (D) 13.3 kV
Shunt Capacitor
Solution:  (D) In this method, capacitors connected in parallel with the load.
2Z L Shunt capacitors are generally used when the bus voltage
The transmitted voltage = ×V less than the specified voltage, i.e., under voltage condition.
Z L + ZS
By connecting shunt capacitors, the VARS power required
2 × 80 by the load decreases so that the reactive power drawn by
= × 40 × 103
80 + 400 the load from the source decreases and the power factor,
= 13.33 kV voltage of the bus Increases.
Consider a power system network as shown in the figure.
Tap-changing Transformer
Load In this method, the voltage of the line changes by chang-
I jx ing the position of the winding connection as shown in the
P + jQ figure.
P
S
VS VR
Figure 16  Power system single-line diagram Q
VS
Generally the load is Inductive load. The complex power at
R VR
the sending end is
S = Vs* I = P – jQ
The above figure shows that the secondary voltage of the
P − jQ transformer changes by changing the corresponding switch
I=
Vs* position.

⎛ P − jQ ⎞ Series Capacitors
The receiving end voltage VR = VS – j ⎜ ⎟x
⎝ Vs ⎠ In this method, a capacitor is connected in series with the
( Vs – reference voltage) line. So, the resultant reactance of the line decreases then
the corresponding voltage drop of the line also decreases.
⎡ Q ⎤ P The change in voltage by connecting series capacitor is
VR = ⎢Vs − X ⎥ − j X
⎣ Vs ⎦ V2 ∆V = IXc sinf.
E1
From the above equation VR is not considerable affect by the
IXC
variation of real power then by the variation reactive power
of the load, Bus voltage is varied. So, by controlling the IXL
reactive power the voltage is maintained within the speci- E2
fied limits.

Methods of Voltage Control V IR

The different voltage control methods are


f
O I
1. Shunt reactors
2. Shunt capacitors In the above phasor diagram E1 represent the sending end
3. Tap-changing transformers voltage before connecting the series capacitor and E2 rep-
4. Synchronous condensers resent the sending end voltage after connecting the series
5. Series capacitors capacitor.
6. Static VAR system Therefore by connecting series capacitor the voltage
drop decreases by IXc factor.
3.894 | Power Systems

Power Factor Improvement OA = VI cosf which represents the active power in watts
(or) kW.
As the electrical energy is generated, transmitted and dis-
AB = VI sinf represents the reactive power in VAR or kVAR.
tributed in the form of alternating wave, hence power factor
OB = VI represents the apparent power in VA or kVA
is considered. As most of the loads are inductive in nature,
therefore they have low lagging power factor. Low power From the power triangle, we can infer that,
factor causes the increase active power loss. To ensure the
1. The apparent power in AC circuit has two components,
most favourable conditions for a supply system from engi-
i.e. active and reactive power at right angles to each
neering and economical standpoint, it is important to have
other.
power factor close to unity.
OB2 = OA2 + AB2
Power Factor
⇒ (Apparent power)2 = (Active power)2 + (Reactive
The cosine of angle between voltage and current in an AC power)2
circuit is known as power factor.
OA Active power
In an AC circuit, there is generally phase difference ‘f’ 2. Power factor cosf = =
between voltage and current. The term cosf is called the OB Apparent power
power factor of the circuit. 3. For leading currents, the power triangle should be
If the circuit is inductive, the current lags the voltage and reversed. To improve the power factor of the load, the
power factor is said to be lagging. In case of capacitive cir- device taking leading reactive power (e.g. capacitor) is
cuit, current leads the voltage and the power factor is said connected in parallel with the load, neutralizing partly
to be leading. the lagging reactive power.
Consider an inductive circuit taking a lagging current ‘I’
form the supply voltage ‘V’ the lagging angle being equal Disadvantages of Low Power Factor
to ‘f’. The vector diagram for the above circuit is shown
We know that PL = VLIL cosf
below.
I cosf P
0 V
⇒ IL = (for single-phase)
f VL cos ϕ
I sinf P = 3VL I L cos ϕ
I
P
⇒ IL = (for three-phase)
The current ‘I’ is resolved into two perpendicular 3VL cos ϕ
components, From the above expression, we can conclude that for the
fixed power and voltage, the load current is inversely pro-
Icosf in phase with ‘V’
1.
portional to the power factor.
Isinf, 90° out of phase with V. (for L and C)
2.
Due to the low power factor, the current drawn from the
Icosf is known as active (or) wattful component while source is large resulting in the following disadvantages.
the component Isinf is called Reactive (or) wattless com-
1. The electrical machinery is always rated in kVA.
ponent. If the reactive component is small, then the phase
While designing the machine, the power factor of the
angle ‘f’ is small and hence the power factor cosf will be
load is not known.
high. Hence the circuit having small reactive current (I sinf)
kW
will have high power factor and vice versa. The value of kVA =
power factor can never be more than unity. cos ϕ
From the above formula, kVA rating of the Equip-
Power Triangle ment is inversely proportional to the power factor.
Hence, lower the power factor, the kVA of the ma-
Power factor can also be analysed by the power drawn by the chinery should be increased.
AC circuit. If each side of the current triangle is multiplied 2. The large current drawn at low power factor causes
by voltage V, then we get the power triangle. more copper losses (I2R) in all the elements of the
VI cosf supply system. Hence the efficiency is reduced.
0 A
f
Economics of Power Factor Improvement
VI sinf By using power factor improvement devices the power fac-
VI tor of the load Increases. Therefore net maximum demand
power drawn from the supply decreases then the corre-
B sponding total current and transmission losses decreases
Chapter 1  Power System Generation, Transmission and Distribution  |  3.895

which leads to the net saving on the economic cost. But by V22
considering the cost and maintain of power factor correc- New case reactive power absorbed =
2π f 2 L
tion equipment the total saving is decreases. Generally P
is the expenditure per kVA per annum of the power factor
Q2 =
( 0.98V1 )
2

correction equipment and q is the rate per kVA of maximum 2π (0.95 f1 ) L


demand per annum. Therefore the corresponding economic
power factor improvement is cosf, i.e., 0.982
Q2 = Q1 = 1.0109Q1
2
0.95
⎛P⎞ Q2 = 101.09 MVAR
cos ϕ = 1 − ⎜ ⎟
⎝q⎠
The power factor is improved by different methods. Example 50:  A balanced delta connected load of (8 + j6) Ω
per phase is connected to a 400 V, 60 Hz, 3-f supply lines. If
1. Static VAR compensator the input power factor is to be improved to 0.9 by connect-
2. Synchronous condenser ing a bank of star connected capacitor, the required kVAR
3. STATCOM of the bank is.
4. Static capacitor (A) 28.8 kVAR (B) 18.5 kVAR
(C) 10.7 kVAR (D) 20.4 kVAR
Static VAR Compensator
In this method, a capacitor bank fixed or switched (or) fixed, Solution:  (C)
capacitor and switch reactor bank are used in parallel. These 400
Per-phase load current = = 40 ∠ − 36.86°
type of compensations are generally used when reactive 8 + j6
power leading and lagging by corresponding switching ON
Load power factor = cos 36.86 = 0.8 lagging
and OFF of the capacitors and reactors into the circuit. By
Capacitor placement affects the reactive component of load
changing the reactive power, the power factor of the circuit
but real power component of the load remains unchanged
also changes.
\ I1 cosf1 = I2 cosf2
Static Capacitor 40 × 0.8 = I2 × 0.9 ⇒ I2 = 35.55 A
In the above method both capacitors and reactors are con-
I2 = I1 + Ic from the system configuration
nected but In these method only capacitors are used in
parallel with the load then there is net improvement of the Ic = I2 – I1 [\f2 = cos–1 0.9]
power factor and also the reactive power drawn from the = 35.55 ∠–25.84 – 40 ∠–36.86
supply also reduces.
Ic = 8.5 ∠90o
Synchronous Condenser Reactive power of capacitor bank
It is a synchronous motor operating under no-load and over = Q at 0.8 power factor – Q at 0.9 power factor
excitation condition. When these motor is connected in par-
allel with the load then the motor supplying reactive power = 3VL1 I L1 sin ϕ1 − 3VL I L2 sin ϕ2
to the load. There for the corresponding power factor of the
load Increases.  = 3 × 400 × 3 × 40 × 0.6 − 3 × 400 × 3 × 35.5 × sin 25
 = 10.7 kVAR
Example 49:  A shunt reactor of 100 MVAR is operated at
98% of its rated voltage and 95% of its rated frequency the Example 51:  At an industrial substation with a 4 MW load,
reactive power absorbed by the reactor is a capacitor of 2.5 MVAR is installed to maintain the load
(A) 98.92 MVAR (B) 101.09 MVAR factor at 0.97 lagging. If the capacitor goes out of service
(C) 95.32 MVAR (D) 104.32 MVAR the load power factor is
(A) 0.9 (B) 0.75
Solution:  (B) (C) 0.8 (D) 0.842
V2
Reactive power absorbed by reactor = Solution:  (B)
XL
V12 Initial phase difference f1 = cos–1 0.97
Q1 = = 100 MVAR = 14.07o
2π f1 L
kVAR supplied by capacitor bank = P(tanf1 – tanf2)
V2 = 0.98 V1
4 × 106 (tanf1 – tan14.07) = 2.5 × 106
f2 = 0.95 f1 f1 = 41.2o
3.896 | Power Systems

Power factor before adding capacitor bank


cosf1 = 0.752 Characteristic impedance α Zseries

Example 52: A loss less transmission line having surge Z C1 Zseries
=
impedance loading of 2000 MW. A series capacitive com- Z C2 0.7 × Zseries
pensation of 30% the surge impedance loading of the com-
pensated transmission line will be Characteristic impedance after compensation
(A) 1673 MW (B) 2390 MW
ZC = 0.836 ZC
(C) 600 MW (D) 2600 MW 2 1

1
Since surge impedance loading α =
Solution:  (B) ZC

V2 SIL1 Z C2
Surge impedance loading (SIL) =
ZC =
SIL 2 Z C1

1
Surge impedance loading α Z C1
ZC SIL 2 = SIL1 ×

Z C2
When the capacitive compensation of 30% is provided, the
= 1.195 × 2000 = 2390 MW
series impedance of the line is reduced to 70% of its original
value which in turn reduced the characteristic impedance.

Exercises
Practice Problems 1 5. The figure shows a single circuit 460 kV line using two
Directions for questions 1 to 27:  Select the correct alterna- bundled conductors per phase. If diameter of each con-
tive from the given choices. ductor is 5 cm, the inductance/km/phase is
80 cm
1. A generating station has a maximum demand of
60 MW. The plant capacity factor is 0.5 and the load
factor is 0.6. The plant running as per schedule is fully
loaded. The daily energy produced will be
(A) 720 MWh (B) 964 MWh
(C) 864 MWh (D) 820 MWh
8.5 m 8.5 m
2. The inductance in mH/km of a single-phase overhead
transmission line of diameter 1 cm and distance of sep- (A) 0.24 mH/km/phase
aration 60 cm is (B) 0.76 mH/km/phase
(A) 0.05 + 0.2 ln 60 (C) 0.89 mH/km/phase
(D) 0.90 mH/km/phase
⎛ 60 ⎞
(B) 0.05 + 0.2 ln ⎜
⎝ 0.5 ⎟⎠ 6. A conductor is composed of seven identical strands of
copper, each having a radius r. The self-GMD of the
⎛ 60 ⎞ conductor will be
(C) 0.2 ln ⎜
⎝ 0.5 ⎟⎠ (A) 2.17 r (B) 2.645 r
(D) 0.2 ln 60 (C) 2.141 r (D) 1.21 r
7. Determine the capacitance for a 2-conductor 1-f line
3. A 50 km transmission line when excited with a source operating at 50 Hz, the conductors being of 2 cm in
of frequency 500 Hz, it will be modelled as diameter and spaced 3 m apart.
(A) Short line (B) Medium line (A) 9.74 × 10–9 F/km (B) 4.87 × 10–9 F/km
(C) Long line (D) Data insufficient (C) 3.05 × 10 F/km –9
(D) None of these
4. An existing AC system has a string efficiency of 80%. 8. A 1-f transmission line having R = 1.4 W and X = 0.8 W
The same line, if used for DC transmission, then string is delivering a load of 200 kVA at 2500 V. The voltage
efficiency will be at sending end when p.f. of the load is 0.8 p.f. lag is
(A) 100% (B) >80% (A) 2.069 kV (B) 2.840 kV
(C) <80% (D) 0% (C) 2.627 kV (D) 3.54 kV
Chapter 1  Power System Generation, Transmission and Distribution  |  3.897

9. The parameters of a single circuit 3-f, 50 Hz transmis- (A) Curve A (B) Curve B
sion line are (C) Curve C (D) Curve D
R = 0.2 W/km, L = 1.3 mH/km 13. A 275 kV transmission line has the following line con-
C = 0.01 mF/km, L = 120 km stants. A = 0.85 ∠5°, B = 200 ∠75°. The power that can
be received at unity p.f. if voltage profile at each end is
The efficiency of the line if it delivers 40 MW at 132
to be maintained at 275 kV is
kV and 0.8 p.f. lag is
(A) 118 MW (B) 218 MW
(A) 85.4% (B) 90.0%
(C) 98 MW (D) None of these
(C) 98% (D) 92.6%
14. In a 3-f, 4-wire system, the line voltage is 400 V and
10. A DC distribution system is shown with load currents
now inductive loads of 10, 8 and 5 kW are connected
as marked. The two ends of the feeder P and Q are fed
between the three line conductors and the neutral as
such that VP – VQ = 3 V. The value of voltage VP for
shown in figure. Calculate the current in the neutral
a minimum voltage of 220 V at any point along the
conductor.
feeder is
R
P 0.15 Ω 0.15 Ω 0.15 Ω Q

10 kW

10 A 20 A 30 A 15 A
(A) 220 V (B) 223 V
8 kW 5 kW
(C) 226 V (D) 229 V
11. An underground cable with a surge impedance of 100 W
B
is connected in series with an overhead line having a
surge impedance of 400 W. If a source of 50 kV travels Y
from the line towards the line cable junctions, the value
of the transmitted voltage wave at the junction is
(A) 20 kV (B) 30 kV (A) 15.8 A (B) 16.8 A
(C) –30 kV (D) 80 kV (C) 17.8 A (D) 18.8 A
12. For the figures given below, which curve has the least 1 5. A 3-f transmission line consists of 1 cm radius con-
load factor? ductors spaced symmetrically 4 m apart. Dielectric
strength of air is 30 kV/cm. The line voltage for corona
(A) 40 MW to commence is
(A) 110 kW (B) 220 kV
Area = (C) 330 kV (D) 400 kV
360 Hours Common Data for Questions 16 and 17:
24
(B) The conductors of a 3-f transmission line are arranged
25 MW
as shown, radius of each line = 0.4 cm

Hours
24 1.6 m 1.6 m

(C)
40 MW 3.2 m

The operating voltage is 132 kV.


Hours
16. The capacitance per phase is
24 (A) 8.9 × 10–9 F/km
(D) (B) 9.2 × 10–6 F/km
40 MW (C) 11.5 × 10–9 F/km
(D) 7.5 × 10–7 F/km
Hours 17. The charging current when the transmission line is
14 24 operated at 264 kV is
3.898 | Power Systems

(A) 0.312 A/km 23. The per-unit impedance of a circuit element is 0.30. If
(B) 0.257 A/km the base kV and base MVA are halved, then the new
(C) 0.427 A/km value of per unit impedance is
(D) 0.214 A/km (A) 0.9 (B) 0.6
(C) 0.3 (D) 0.15
Common Data for Questions 18 and 19:
24. Two buses 1 and 2 are connected by an impedance (0 +
A 3-f 50 Hz transmission line is 400 km long. The volt-
j10) W. The bus 1 voltage is 400∠45 and bus 2 voltage
age at the sending end is 220 kV. The line parameters
is 400∠0°V. The real and reactive powers supplied by
are r = 0.125 W/km, x = 0.4 W/km, y = 2.8 × 10–6 /km
bus 1, respectively, are
18. The sending end current and receiving end voltage are. (A) +11314 W, +4686 VAR
(A) 76 A, 121 kV (B) 152 A, 242 kV (B) +11314 W, –4686 VAR
(C) 152 A, 226 kV (D) 76 A, 242 kV (C) –11314 W, –2687 VAR
19. The maximum permissible line length of receiving end (D) –5656 W, +2687 VAR
no load voltage is not to exceed 235 kV 25. A load is drawing 500 kW at 0.707 lag. A synchronous
(A) 138 km (B) 238 km motor improves the p.f. of the load to 0.96. The motor
(C) 338 km (D) 438 km carries a load of 300 kW. The leading reactive power
20. A 100 MVA, 11 kV alternator has synchronous reac- supplied by the motor is
tance 0.3 p.u. The p.u. synchronous reactance on a (A) 500 kVAR (B) 233.33 kVAR
50 MVA, 33 kV base is (C) 266.67 kVAR (D) 250 kVAR
(A) 1.67 p.u. (B) 0.45 p.u. 26. The voltage phasor of a circuit is 20∠15°V and current
(C) 0.016 p.u. (D) 0.167 p.u. phasor is 5∠–45°A. The active and reactive powers in
21. A 3-f induction motor rated at 400 V, 50 Hz is running the circuit are
at a p.f. of 0.707 lag. The input to the motor is 8 kVA. It (A) 50 W and –86.67 VAR
is proposed to improve the p.f. of the system to 0.9 by (B) 50 W and 86.67 VAR
a D connected bank of capacitors. The value of capaci- (C) 86.67 W and 50 VAR
tance/phase required is (D) 86.67 W and –50 VAR
(A) 0.58 mF (B) 58 mF 27. A 240 V single-phase AC source is connected to a load
(C) 0.19 mF (D) 19 mF with an impedance of 20∠45. A capacitor is connected
22. A shunt capacitor rated 100 MVAR is operated at 99% in parallel with the load. If the capacitor supplies
of rated voltage and 98% of rated frequency. The reac- 2400 VAR, the real power supplied by source is
tive power absorbed would be approximately (A) 3394 W (B) 2400 W
(A) 100 MVAR (B) 98 MVAR (C) 994 W (D) None of these
(C) 96 MVAR (D) 95 MVAR

Practice Problems 2 3. A two conductor 1-f line operating at 50 Hz has a


Directions for questions 1 to 40:  Select the correct alterna- diameter of 20 mm and spacing between the conduc-
tive from the given choices. tors is 3 m. Height of conductor above the ground is
6 m. The capacitance of line to neutral will be
1. A 1-f two conductor circuit has a spacing of 3 m
(A) 9.7 pF/km (B) 10.2 pF/km
between them and each conductor has a diameter of
(C) 8.7 mF/km (D) 2.4 mF/km
10 mm. The inductance of each circuit per km is
(A) 1.24 mH/km (B) 1.19 mH/km 4. The power losses in a 3-f 20 km line delivering 10 MW
(C) 2.38 mH/km (D) 0.59 mH/km at 11 kV, 50 Hz, 0.8 p.f. lagging is 10% of power deliv-
ered. The resistance of each conductor is
2. The conductors of a 3-f 50 Hz circuit are each of diam-
(A) 0.774 W (B) 2.14 W
eter 21 mm. The spacing is as shown. The inductance is
(C) 1.54 W (D) 0.86 W
5. If bundled conductors are used instead of single
3m 3.6 m ­conductors, then
(A) Inductance and capacitance increase
3m (B) Inductance and capacitance decrease
(A) 10.5 × 10 H/km
–4
(B) 8.2 × 10–4 H/km (C) Inductance increases and capacitance decreases
(C) 5.6 × 10–3 H/km (D) None of these (D) Inductance decreases and capacitance increases
Chapter 1  Power System Generation, Transmission and Distribution  |  3.899

6. A 400 V, 3-f 4-wire system supplies a Y-connected 13. The transmission capacity of a line is related to its
load. Resistance of each line is 0.2 W and of neutral is inductance varies in
0.4 W. The load impedances are (A) Direct proportion
ZR = 6 + j10, ZY = 10 W (B) Inverse proportion
ZB = 8 – j10 W. Current is neutral is (C) Square
(A) 16.56 A (B) 17.56 A (D) Independent of inductance
(C) 18.56 A (D) 19.56 A 14. A lossless radial transmission line with surge imped-
7. The non-uniform distribution of voltage across the ance loading
units in a string of suspension-type insulator is due to (A) Has sending end voltage higher than receiving end
(A) Non-uniform distance between the cross arm and voltage
the units (B) Has flat voltage profile and unity p.f. at all points
(B) Existence of the stray capacitance between the me- along it
tallic junctions of the units and tower body (C) Takes positive VAR at sending end and zero VAR
(C) Non-uniform distance of separation of units from at receiving end
tower body (D) Takes negative VAR at sending end and zero VAR
(D) Unequal self-capacitance of the units at receiving end
8. A long overhead transmission line is terminated by its 15. The surge impedance of a 400 km line is 500 W. For a
characteristic impedance. The ratio of voltage to cur- 200 km length of same line, the impedance will be
rent at different points along the line (A) 250 W (B) 1000 W
(A) Progressively increases from receiving end to the (C) 500 W (D) 750 W
sending end 16. In a string of suspension insulators, the voltage distri-
(B) Progressively decreases from receiving end to the bution across the different units could be made uniform
sending end by using grading ring because it
(C) Remains the same at the two ends being maximum (A)  Decreases the capacitances of upper insulators
at the centre of the line units causing equal voltage drop
(D) Remains the same at all points (B) Increases capacitances of lower insulator units to
9. A 1-f, AC distributor supplies two single-phase loads cause equal voltage drop
as shown in the figure. (C) Forms capacitances with link pins which helps to
cancel the charging current from link pins
A B C
(D) Forms capacitances with link pins to carry the
(0.01 + j 0.08) Ω (0.08 + j 0.12) Ω 400 V
charging current from the link pins
60 A 120 A 17. The cause of Ferranti effect in transmission lines is
at 0.8 lag at 0.8 lag (A) Line inductance
The voltage drop from A to C is (B) Line capacitance
(A) 21 V (B) 26.7 V (C) Line capacitance and inductance
(C) 41 V (D) 51 V (D) Line resistances an capacitance
10. The use of hollow conductors reduce corona loss as 18. Series capacitors are used in a transmission line to
(A) Provides better ventilation of the conductor (A) To increase the phase angles at sending end and
(B) Eddy currents in the conductor are eliminated receiving ends under heavy load conditions
(C) Current density is reduced (B) Reduce the phase angle between sending end and
(D) For a given cross section radius of conductor is receiving ends under heavy load conditions
­increases (C) To reduce the capacitive current
(D) None of these
11. An insulator string has three units. The capacitance
of the link between two units to earth is one eight of 19. The load factor and capacity factor of a plant are
the capacitance of each insulator. Each unit has a safe equal if
working voltage of 18.5 kV. The system voltage is (A) Peak load is equal to capacity of plant
(A) 55.5 kV (B) 46.7 kV (B) Average load is half the capacity of the plant
(C) 37 kV (D) 36.7 kV (C) Average load is same as peak load
(D) Group diversity factor is equal to peak diversity
12. The increase in resistance of a conductor carrying factor
alternating current, due to non-uniform distribution of
current is called 20. The voltage across the units in a two unit suspension
(A) Hall effect (B) Proximity effect insulator is 40% and 60%. The ratio of capacitance of
(C) Skin effect (D) Ferranti effect insulator to that of earth will be
3.900 | Power Systems

(A) 0.4 (B) 0.45 30. Shunt reactors are connected in transmission lines to
(C) 0.5 (D) 0.55 (A) Limit the fault current
2 1. The receiving end voltage of a long transmission line at (B) Improve stability limit
no load condition is (C) Absorb the excess reactive power
(A) Less than sending end voltage (D) Generate requisite reactive power
(B) Greater than sending end voltage 31. The excitation of a synchronous generator connected to
(C) Equal to sending end voltage an infinite bus bar delivering power at a lagging power
(D) Zero voltage factor is increased. Then
22. The level of insulation of a 400 kV extra high voltage (A) Current delivered increases
line is decided based on (B) Terminal voltage increases
(A) Lightning over voltage (C) Voltage angle d increases
(B) Switching over voltage (D) Both A and C
(C) Corona inception voltage 32. The voltage of an infinite bus bar is 1∠0° and that of
(D) Radio and TV interference an alternator connected to it is 0.95∠0°. The alternator
23. The transmission line feeding power on either side of acts as a
main transmission line is (A) Shunt coil
(A) Primary distribution (B) Shunt capacitor
(B) Primary transmission (C) Shunt resistance
(C) Secondary transmission (D) None of these
(D) Secondary distribution 33. The power dispatch through a transmission line can be
24. Suppose the conductors are dead ended and there is a increased by installing
change in the direction of transmission line, insulators (A) Shunt capacitance
used are (B) Series capacitance
(A) Strain type (B) Pin type (C) Shunt reactance
(C) Shackle type (D) Suspension type (D) Series reactance
25. For a fixed inductive shunt compensation, series capac- 34. The combined frequency regulation of machines in an
itive compensation area of capacity 3000 MW and operating at a normal
(A) Increases virtual surge impedance loading frequency of 60 Hz is 0.15 p.u. The regulation in Hz/
(B) Decreases vertical surge impedance loading MW is
(C) Increases effective length of line 3 0.3
(D) None of these (A) (B)
1000 1000
Common Data for Questions 26 and 27: 1 1
(C) (D)
A 50 Hz, 3-f, transmission line 30 km long has a total series 200 100
impedance of (40 + j125) W and shunt admittance of 10–3
35. Which amongst the following is not true?
mho. The load is 50 MW at 220 kV with 0.8 lagging p.f.
(A) Synchronous phase modifiers are installed at send-
26. Sending end voltage using nominal p is ing end
(A) 216 kV (B) 204 kV (B) Synchronous phase modifiers are installed at load
(C) 232 kV (D) 238 kV end
27. The sending end power factor is (C)  Synchronous phase modifiers are specially de-
(A) 0.8 lag (B) 0.9 lag signed synchronous motors
(C) 0.98 lag (D) 1 (D) Synchronous phase modifiers do not carry load
28. A pure inductance absorbs reactive power from the AC 36. In EHV lines, series capacitance compensation is used
line when to
(A) Both applied voltage and current decreases (A) Improve the stability of the system
(B) Both applied voltage and current increases (B) Reduce the loading on the line
(C) Applied voltage increases and current decreases (C) Improve the protection of the line
(D) Applied voltage decreases and current increases (D) Reduce the voltage profile

29. The power factor of the load is corrected by connecting II.  Question Nos.: 37 and 38
a static capacitor in parallel with the load. The power A line operating at 60 Hz is 81% series compensated.
taken by the load 37. The resonating frequency is
(A) Increases (B) Decreases (A) 60 Hz (B) 66.6 Hz
(C) Remains same (D) Is uncertain (C) 50 Hz (D) 54 Hz
Chapter 1  Power System Generation, Transmission and Distribution  |  3.901

38. Now if the line is 100% series compensated. Resonant 39. The p.u. value of impedance when referred to the LV
frequency is side is
(A) 60 Hz (B) 66.6 Hz (A) 2.13∠32° (B) 0.0085∠62°
(C) 50 Hz (D) 54 Hz. (C) 0.085∠62° (D) 0.213∠32°
III.  Question Nos.: 39 and 40 40. If the transformer has an exciting current of 10 A on
A20 kVA, 2000/200 V single-phase transformer has an HV side, p.u. value of current referred to LV side is
equivalent impedance of Z = 8 + j15 when referred to the (A) 1 p.u. (B) 2 p.u.
HV side. (C) 0.5 p.u. (D) 0.25 p.u.

Previous Years’ Questions


1. The concept of an electrically short, medium, and (A) 122 km (B) 172 km
long line is primarily based on the [2006] (C) 222 km (D) 272 km
(A) nominal voltage of the line 6. Out of the following plant categories
(B) physical length of the line
(i) Nuclear (ii)  Run-of-river
(C) wavelength of the line
(D) power transmitted over the line (iii) Pump storage (iv)  Diesel
2. A 400 V, 50 Hz, three-phase balanced source supplies The base load power plants are [2009]
power to a star connected load whose rating is 12 3 (A) (i) and (ii) (B) (ii) and (iii)
kVA, 0.8 p.f. (lag). The rating (in kVAR) of the delta (C) (i), (ii) and (iv) (D) (i), (iii) and (iv)
connected (capacitive) reactive power bank necessary 7. For a fixed value of complex power flow in a trans-
to bring the p.f. to unity is [2006] mission line having a sending end voltage V, the real
(A) 28.78 (B) 21.60 power loss will be proportional to [2009]
(C) 16.60 (D) 12.47 (A) V (B) V2
1
3. The A, B, C, D constants of a 220 kV line are: (C) 2 (D) 1/V
V
A = D = 0.94 ∠1°, B = 130∠73°, C = 0.001∠90°. If
the sending end voltage of the line for a given load 8. Match the items in List-I with the items in List-II and
delivered at nominal voltage is 240 kV, then % volt- select the correct answer using the codes given below
age regulation of the line is [2006] the lists. [2009]
(A) 5 (B) 9
(C) 16 (D) 21 List I List II
To Use
4. Consider a bundled conductor of an overhead line,
consisting of three identical sub-conductors placed a.  Improve power factor 1.  Shunt reactor
at the corners of an equilateral triangle as shown b.  Reduce the current ripples 2.  Shunt capacitor
in the figure. If we neglect the charges on the other c. Increase the power flow in 3.  Series capacitor
phase conductors and ground, and assume that spac- line
ing between sub-conductors is much larger than their
radius, the maximum electric field intensity is experi- d.  Reduce the Ferranti effect 4.  Series reactor
enced at [2007] (A) a → 2 b → 3 c → 4 d → 1
Y (B) a → 2 b → 4 c → 3 d → 1
(C) a → 4 b → 3 c → 1 d → 2
(D) a → 4 b → 1 c → 3 d → 2
X
9. Match the items in List-I with the items in List-II and
Z W select the correct answer using the codes given below
the lists. [2009]

(A) Point X (B) Point Y List I List II


(C) Point Z (D) Point W Type of transmission Type of distance relay
line preferred
5. The total reactance and total susceptance of a lossless
overhead EHV line, operating at 50 Hz, are given by a.  Short line 1.  Ohm relay
0.045 p.u. and 1.2 p.u., respectively. If the velocity of b.  Medium line 2.  Reactance relay
wave propagation is 3 × 105 km/s, then the approxi- c.  Long line 3.  Mho relay
mate length of the line is[2007]
3.902 | Power Systems

(A) a → 2 b → 1 c → 3 13. Consider a three-phase, 50 Hz, 11 kV distribution sys-


(B) a → 3 b → 2 c → 1 tem. Each of the conductors is suspended by an insu-
(C) a → 1 b → 2 c → 3 lator string having two identical porcelain insulators.
(D) a → 1 b → 3 c → 2 The self-capacitance of the insulator is 5 times the
shunt capacitance between the link and the ground, as
10. Three generators are feeding a load of 100 MW. The shown in the figure. The voltage across the two insula-
details of the generators are [2009] tors is [2010]
Rating Efficiency Regulation
(MW) (%) (p.u.) on 100
e2
MVA base 5C

Generator-1 100 20 0.02


C 5C
Generator-2 100 30 0.04 e1
Generator-3 100 40 0.03
Conductor
In the event of increased load power demand, which
of the following will happen? (A) e1 = 3.74 kV, e2 = 2.61 kV
(A) All the generators will share equal power (B) e1 = 3.46 kV, e2 = 2.89 kV
(B) Generator-3 will share more power compared to (C) e1 = 6.0 kV, e2 = 4.23 kV
Generator-1 (D) e1 = 5.5 kV, e2 = 5.5 kV
(C) Generator-1 will share more power compared to 14. Consider a three-core, three-phase, 50 Hz, 11 kV
Generator-2 cable whose conductors are denoted as R, Y and B in
(D) Generator-2 will share more power compared to the figure. The inter-phase capacitance (C1) between
Generator-3 each pair of conductors is 0.2 mF and the capacitance
between each line conductor and the sheath is 0.4 mF.
11. An extra high voltage transmission line of length The per-phase charging current is [2010]
300 km can be approximated by a lossless line hav-
ing propagation constant b = 0.00127 radians per km.
C2
Then the percentage ratio of line length to wavelength
will be given by [2008] R
(A) 24.24% (B) 12.12% C1 C1
(C) 19.05% (D) 6.06%
B Y
12. Single line diagram of a 4-bus single source distribu- C1 C2
C2
tion system is shown below, branches e1, e2, e3 and e4
have equal impedances. The load current values indi-
Outer sheath
cated in the figure are in per unit. Distribution com-
(A) 2.0 A
pany’s policy requires radial system operation with (B) 2.4 A
(C) 2.7 A
minimum loss. This can be achieved by opening of (D) 3.5 A
the branch 1 5. For enhancing the power transmission in a long EHV
[2008]
transmission line, the most preferred method is to
connect a[2011]
(A) Series inductive compensator in the line.
e1 e2
(B)  Shunt inductive compensator at the receiving
end.
(C) Series capacitive compensator in the line.
(D) Shunt capacitive compensator at the sending end.
1+j 0 e3 e4 5+j 0
16. A nuclear power station of 500 MW capacity is
located at 300 km away from a load centre. So the
2+j0 most suitable power evacuation transmission configu-
(A) e1 (B) e2 ration among the following options. [2011]

(C) e 3
(D) e 4
Chapter 1  Power System Generation, Transmission and Distribution  |  3.903

Load phasors during the fault (on a common reference),


(A) center after the natural transients have died down, are also
132 KV, 300 km
double circuit shown. Where is the fault located? [2015]
I 1 Transmission Line I3
Load V1
(B) center
V2
Q S
∼ ∼
132 KV, 300 km single circuit with P Transmission Line
40% series capacitor compensation R
I2 I4
V2
Load
(C) center V1
400 KV, 300 km I3
I2 I1
single circuit
Load I4
(D) center
400 KV, 300 km (A) Location P
double circuit (B) Location Q
(C) Location R
17. A distribution feeder of 1 km length having resist- (D) Location S
ance, but negligible reactance, is fed from both the
21. A composite conductor consists of three conductors
ends by 400 V, 50 Hz balanced sources. Both voltage
of radius R each. The conductors are arranged as
sources S1 and S2 are in phase. The feeder supplies
shown below. The geometric mean radius (GMR) (in
concentrated loads of unity power factor as shown in
cm) of the composite conductor is kR. The value of k
the figure. [2014]
is ______. [2015]
S1 S2
400 m 200 m P 200 m 200 m 3R

400 V 400 V R
200 A 100 A 200 A 60°
50 Hz 50 Hz

The contributions of S1 and S2 in 100 A current sup-


plied at location P, respectively, are
60°
(A) 75 A and 25 A
(B) 50 A and 50 A
(C) 25 A and 75 A
(D) 0 A and 100 A
18. Shunt reactors are sometimes used in high-voltage 22. A three-phase cable is supplying 800 kW and 600
transmission systems to[2004] kVAr to an inductive load. It is intended to supply an
(A) limit the short-circuit current through the line additional resistive load of 100 kW through the same
(B) compensate for the series reactance of the line cable without increasing the heat dissipation in the
under heavily loaded condition. cable, by providing a three-phase bank of capacitors
(C) limit overvoltages at the load side under lightly connected in star across the load. Given the line volt-
loaded condition. age is 3.3 kV, 50 Hz, the capacitance per phase of the
(D) compensate for the voltage drop in the line under bank, expressed in microfarads, is ______.[2016]
heavily loaded condition. 23. Single-phase transmission line has two conductors
19. Base load power plants are [2015] each of 10 mm radius. These are fixed at a center-
P: wind farms. to-center distance of 1 m in a horizontal plane. This
Q: run-of-river plants. is now converted to a three-phase transmission line
R: nuclear power plants. by introducing a third conductor of the same radius.
S: diesel power plants. This conductor is fixed at an equal distance D from
(A) P, Q and S only the two single-phase conductors. The three-phase line
(B) P, R and S only is fully transposed. The positive sequence inductance
(C) P, Q and R only per phase of the three-phase system is to be 5% more
(D) Q and R only than that of the inductance per conductor of the sin-
20. A sustained three-phase fault occurs in the power gle-phase system. The distance D, in meters, is _____.
system shown in the figure. The current and voltage [2016]
3.904 | Power Systems

24. The inductance and capacitance of a 400kV, three- 25. At no load condition, a 3-phase, 50Hz, lossless power
phases 50HZ lossless transmission line are 1.6mH/ transmission line has sending-end and receiving-
km/phase and 10nF/km/phase respectively. The send- end voltages of 400KV and 420KV respectively.
ing end voltage is maintained at 400kV. To maintain a Assuming the velocity of travelling wave to be the
voltage of 400kV at the receiving end, when the line velocity of light, the length of the line, in km, is
is delivering 300MW load, the shunt compensation ________. [2016]
required is  [2016] 26. The power consumption of an industry is 500kVA, at
(A) Capacitive (B) Inductive 0.8 p.f lagging. A synchronous motor is added to raise
(C) Resistive (D) Zero the power factor of the industry to unity. If the power
intake of the motor is 100kW, the p. f. of the motor is
________. [2016]

Answer Keys

Exercises
Practice Problems 1
1. C 2. B 3. C 4. A 5. C 6. A 7. B 8. C 9. D 10. C
11. A 12. A 13. A 14. D 15. B 16. A 17. C 18. B 19. C 20. C
21. D 22. C 23. B 24. A 25. C 26. B 27. A

Practice Problems 2
1. B 2. A 3. A 4. A 5. D 6. C 7. B 8. D 9. B 10. D
11. B 12. C 13. B 14. B 15. C 16. C 17. C 18. B 19. A 20. C
21. B 22. B 23. C 24. A 25. D 26. D 27. C 28. A 29. C 30. C
31. D 32. A 33. B 34. A 35. A 36. A 37. D 38. A 39. C 40. A

Previous Years’ Questions


1. B 2. D 3. C 4. B 5. C 6. A 7. C 8. B 9. A 10. C
11. D 12. D 13. B 14. A 15. C 16. D 17. D 18. C 19. C 20. B
21.  1.85 to 1.95 22. 48.23 23. 1.42 24. B 25. 294.847Km 26. 0.316

You might also like